You are on page 1of 143

Galgotias Business School

Greater Noida

Student Handbook

2012

A GBS Faculty Council Initiative




P
a
g
e
2

TABLE OF CONTENTS


Subject Page No.

General Questions 3
Marketing Management 6
Marketing Research 13
Marketing Metrics 24
Rural Marketing 28
Consumer Behaviour 30
Brands & Integrated Marketing Communications 32
Retail Marketing 34
CRM 36
Entrepreneurship and Innovation 39
B2B Marketing 42
Finance 44
Glossary of Finance 52
International Business 59
Supply Chain Management 62
General Questions on Technology 67
General Information on Technology 68
Legal Aspects of Business 70
Economics 72
Quantitative Techniques 81
Operations Research 88
Human Resource Management 94
Leadership and Teambuilding 106
Organization Behaviour II 110
Industrial Relations 114
Cross Cultural Management 115
Negotiation and Counselling Skills 130
Compensation and reward management 135

P
a
g
e
3

General Questions

Interview Question: How would your friends describe you?
Answer Guide: Answer should show focused and hard-working qualities and
competencies. Descriptions should show the skills needed for this position.

Interview Question: Who is your role model?
Answer Guide: Applicants role model should contain the qualities and
competencies employers would like to see in a successful candidate.

Interview Question: What would you like to be doing five years from now?
Answer Guide: Applicant should see themselves well integrated in the company.
Candidate should have researched the company and explain their potential skills
that could benefit the company.

Interview Question: How would you react if I told you your interview so far was
terrible?
Answer Guide: This is a test to see how well the candidate can hold themselves
together. Interviewer is looking for the ability to think on your feet and the skills
to respond articulately under pressure. Applicant should be diplomatic notice that
the interviewer used the word if.

Interview Question: Why should I hire you?
Answer Guide: Candidate should take the opportunity to sell their skills,
knowledge, and abilities. Candidate should try to prove why t hey are perfect for
the job, that they will do the job, and that they are manageable.

Interview Question: How long will you stay with the company?
Answer Guide: Candidate should have researched the company and be able to
explain how he/she will integrate themselves in the company.

Interview Question: What can you do for us that someone else cannot do?
Answer Guide: Candidate should be eager to show his/her knowledge and interest
in the company and eager to show that they have all the necessary skills and
competencies to do the job successfully.

P
a
g
e
4


Interview Question: What was the last book you read or the last movie you saw
and how did it affect you?
Answer Guide: Job seeker should emphasize his/her acquired skills,
competencies, and interests. Some books or videos could demonstrate that the
applicant is trying to improve their skills and knowledge.

Interview Question: Can you tell me what things really bother you?
Answer Guide: This question is similar to others in this section. The want to see if
your answers maintain a reasonable level of consistency or if they are merely a
faade. They are looking out for candidates with chips on their shoulder and
repressed anger. The employer is seeking those who dislike under-performance and
sloppy work practice.

Interview Question: Do you feel you could have done a better job than your
previous boss?
Answer Guide: Candidates should not strongly criticize their old bosses. Their
answers need to show diplomacy and tact with reasonable arguments. An
enterprising attitude could be a positive option.

Interview Question: What are some of the things about your boss that you
disliked?
Answer Guide: Candidate should remain calm and poised. They should not have
strong, unreasonable emotions.

Interview Question: As we still have some time left, can you tell me a story?
Answer Guide: Candidate should maintain his/her composure and use the
question to sell his/her abilities and skills to the employer.




Interview Question: Could I ask you, how do you rate me as an interviewer?

P
a
g
e
5

Answer Guide: This is a test looking for the candidates composure and ability to
think on his/her feet. Applicant should be able to respond articulately under
pressure.



Interview Question: What is the worst thing you have heard about our company?
Answer Guide: This question is designed to shock the applicant and test their
composure and ability to think on their feet


















P
a
g
e
6

Questions on Marketing
1. Explain the concept of segmentation & positioning?
Answer : Segmentation is a process of identifying distinct groups of buyers
who differ in their needs and preferences to homogeneous groups.
Positioning is the act of designing the companys offering and image to
occupy a distinctive place in the mind of the target market.

2. What is the difference between sales and marketing?
Answer : Marketing is a bigger umbrella & sales is part of marketing.
Function of sales deptt. is to achieve numbers in terms of units sold whereas
marketing deptt. has to take care of all Ps.

3. Define marketing and three basic concepts of marketing?
Answer : Marketing is a process by which companies create value for
customers and build strong customer relationships in order to capture value
from customer in return. Three basic concepts are NEEDS (state of felt
deprivation), WANTS (needs pointed in a particular direction become
wants) and DEMANDS (wants backed by buying power).

4. In marketing what is macro & micro environment?
Answer : The forces outside the direct influence of marketing deptt. form a
environment. The actors close to company like suppliers, dealers,
competitors etc form micro environment. The larger societal forces like
political, economical, social & technological forces form macro
environment.

5. What is viral marketing?
Answer : Internet form of word of mouth marketing is viral marketing.

6. What do you mean by brick companies, click companies & brick &
click companies?
Answer : Brick companies are the companies operating with proper office
infrastructure accessible to customer, whereas click companies are

P
a
g
e
7

accessible to customers only on internet and lastly brick & click companies
are combination of both.

7. How would you achieve growth in such a competitive market?
Answer : Can link your answer to Ansoff Matrix.
8. What is the difference between modern trade and traditional trade?
Give examples.
Answer : Traditional trade means selling products through dealers and
distributors, whereas modern trade is selling products through retail chains
like Reliance, Croma, BigBazar, Walmart etc depending on the category
product belongs to.

9. What is the difference between a product and a brand?
Answer : A product is anything we can offer to a market for attention,
acquisition, use, or consumption that might satisfy a need or want. A product
may be a physical good, a service, a retail outlet, a person, an organization, a
place, or even an idea.
While a brand is a name, term, sign, symbol, or design, or a combination of
them, intended to identify the goods and services of one seller or group of
sellers and to differentiate them from those of competition. Many practicing
managers refer to a brand as more than that-as something that has actually
created a certain amount of awareness, reputation, prominence, and so on in
the marketplace.

10. What is the difference between primary and secondary sales?
Answer : Primary Sales means when the organization sells or makes flow of
product or service to the channel partners. E.g. in FMCG : F&C agents,
distributors , dealers etc, in FINANCIAL SERVICE : business partners,
franchisees.
Secondary sales means when the organizations sell or make flow of product
or service to end users. eg : in FMCG : retailers , trade fairs , promotional
sales etc, in FINANCIAL SERVICES : sales directly by company
employees.


P
a
g
e
8

11) What is segmentation?
Segmentation is to identify and profile distinct groups of buyers who differ in their
needs and preferences in to homogeneous groups.

12) What is positioning?
It is the act of designing the companys offering and image to occupy a distinctive
place in the mind of the target market.

13) How will you define Marketing?
Marketing is understanding customer needs and fulfilling them profitably. It can
also be defined as an organizational function and a set of processes for creating,
communicating, and delivering value to customers and for managing customer
relationships in ways that benefit the organization and its stakeholders.

14) What is marketing mix?
The set of controllable, tactical marketing tools that the firm blends to produce the
response it wants in the target market. It consists of Product, Price, Place and
Promotion.

15) What is Marketing Myopia?
Marketing myopia means defining business by the product you sell rather than by
what your customer needs. A myopic marketer pays more attention to the specific
products they offer than to the benefits and experiences produced by the products.

16) What is the difference between primary and secondary sales?
Primary Sales means when the organization sells or makes flow of product or
service to the channel partners. Example IN FMCG : F&C AGENTS,
DISTRIBUTORS , DEALERSEX : IN FINANCIAL SERVICE : BUSINESS
PARTNERS, FRANCHISEES,
Secondary sales means when the organization sell or make flow of product or
service to eh end users. Example : IN FMCG : RETAILERS , TRADE FAIRS ,
PROMOTIONAL SALES EX : FINANCIAL SERVICES : SALES DIRECTLY
BY COMPANY EMPLOYEES



P
a
g
e
9

17) What do you understand Socio-economic classification?
In addition to income classification and consumer classification, Indian households
can also be segmented according to the occupation and education levels of the
chief earner of the household (the person who contributes most to the household
expenses). This is called as Socio-Economic Classification (SEC), which is mainly
used by market planners to target market before launching their new products. SEC
is made to understand the purchase behavior and the consumption pattern of the
households. The urban area is segregated into: A1, A2, B1, B2, C, D, E1, E2.

18) What is the difference between a product and a brand?
A product is anything we can offer to a market for attention, acquisition, use, or
consumption that might satisfy a need or want. A product may be a physical good,
a service, a retail outlet, a person, an organization, a place, or even an idea.
While a brand is a name, term, sign, symbol, or design, or a combination of them,
intended to identify the goods and services of one seller or group of sellers and to
differentiate them from those of competition. Many practicing managers refer to a
brand as more than that-as something that has actually created a certain amount of
awareness, reputation, prominence, and so on in the marketplace.

19) What is brand image?
It is the impression in the consumers' mind of a brand's total personality (real and
imaginary qualities and shortcomings). Brand image is developed over time
through advertising campaigns with a consistent theme, and is authenticated
through the consumers' direct experience.

20) What are the latest trends in marketing?
Green Marketing, Rural Marketing, Modern Retail, Digital Marketing, Social
Media Marketing, Affiliate Marketing, Grass roots Marketing etc.

21) What is network marketing?
Network marketing commonly known as Multi-level marketing (MLM) and
referral marketing, is a term that describes a marketing structure used by some
companies as part of their overall marketing strategy. The structure is designed to
create a marketing and sales force by compensating promoters of company
products not only for sales they personally generate, but also for the sales of other

P
a
g
e
1
0

promoters they introduce to the company, creating a downline of distributors and a
hierarchy of multiple levels of compensation.

22) What is a service encounter/moment of truth in service delivery?
A service encounter is a period of time during which customer interact directly
with a service. It is also called as "Moment of Truth". Service encounters are
transactional interactions in which one person (e.g., a vendor, office clerk, travel
agent) provides a service or good (e.g., a product, an appointment, airline tickets)
to another person.

23) Define Need, Want and Demand:
Need: basic human requirements; a state of felt deprivation eg. Hunger, shelter etc
Want: a specific object that can satisfy a need eg. McDonalds Burger
Demands: wants for specific products backed by an ability to pay

24) Differentiate between Push and Pull Strategy
Push strategy
- Sales force and trade promotion to push the product through the channels.
- Producer promotes product to wholesalers, the wholesalers promote to the
retailers and retailers promote to the consumers.

Pull strategy
- Promotion strategy that spends a lot on advertising and consumer promotion
to create consumer demand.

25) What are Porters generic competitive strategies?
Cost leadership-This strategy involves the firm winning market share by appealing
to cost-conscious or price-sensitive customers. This is achieved by having the
lowest prices in the target market segment, or at least the lowest price to value ratio
(price compared to what customers receive).
Differentiation- Differentiation is aimed at the broad market that involves the
creation of a product or services that is perceived throughout its industry as unique.
The company or business unit may then charge a premium for its product. This

P
a
g
e
1
1

specialty can be associated with design, brand image, technology, features, dealers,
network, or customers service.
Focus-This is not a separate strategy per se, but describes the scope over which the
company should compete based on cost leadership or differentiation. The firm can
choose to compete in the mass market (like Wal-Mart) with a broad scope, or in a
defined, focused market segment with a narrow scope. In either case, the basis of
competition will still be either cost leadership or differentiation.

16) Explain market penetration
Market Penetration: Market penetration occurs when a company enters/penetrates a
market with current products. The best way to achieve this is by gaining
competitors' customers (part of their market share). Other ways include attracting
non-users of your product or convincing current clients to use more of your
product/service (by advertising etc).
17) Define Value and Value Chain
Value is the customers perception of benefits received in relation to the
competition .
Value Chain: A value chain is a chain of activities for a firm operating in a specific
industry. The value chain categorizes the generic value-adding activities of an
organization. The "primary activities" include: inbound logistics, operations
(production), outbound logistics, marketing and sales (demand), and services
(maintenance). The "support activities" include: administrative infrastructure
management, human resource management, technology (R&D), and procurement.
The costs and value drivers are identified for each value activity.

18) What is Below the line communication?
Below the line communication doesnt involve the purchasing of media airtime
or space. This includes sales promotions, direct marketing, channel support
programs, customer and employee communications, sponsorships, web marketing,
sales videos and so on.




P
a
g
e
1
2

19) Discuss promotion mix means?

20) Differentiate between POP and POD
POP and POD: These are utilized in the positioning (marketing) of a brand for
competitive advantage via brand/product:
Points-of-difference (PODs) Attributes or benefits consumers strongly associate
with a brand, positively evaluate and believe they could not find to the same extent
with a competing brand i.e. points where you are claiming superiority or
exclusiveness over other products in the category.
Points-of-parity (POPs) Associations that are not necessarily unique to the brand
but may be shared by other brands i.e. where you can at least match the
competitors claimed benefits. While POPs may usually not be the reason to choose
a brand, their absence can certainly be a reason to drop a brand.

21) What is direct marketing, interactive marketing, ambush marketing
Direct Marketing: It is a form of advertising that reaches its audience without
using traditional formal channels of advertising, such as TV, newspapers or radio.
Businesses communicate straight to the consumer with advertising techniques such
as fliers, catalogue distribution, promotional letters, and street advertising.


P
a
g
e
1
3

Interactive Marketing: It refers to the evolving trend in marketing whereby
marketing has moved from a transaction-based effort to a conversation. Interactive
Marketing allows customers to participate in a Brand's process to build its image in
a certain market or target group's minds.

Ambush Marketing: It is a marketing campaign that takes place around an event
but does not involve payment of a sponsorship fee to the event. For most events of
any significance, one brand will pay to become the exclusive and official sponsor
of the event in a particular category or categories, and this exclusivity creates a
problem for one or more other brands. Those other brands then find ways to
promote themselves in connection with the same event, without paying the
sponsorship fee and without breaking any laws.

22) What do you mean by market share?
Sales of a company or product as a percentage of total sales of that category of
products in a defined area (i.e., a single store, a town, state, country, etc.).



Questions on Marketing Research

1. List the skills required for a successful career in marketing research.
i. Good understanding of marketing principles.
ii. Strong background in statistics and quantitative methods.
iii. Ability to use mainframe, microcomputers, and the Internet.
iv. Solid understanding of the behavioral sciences.
v. Effective verbal and written communication skills.
vi. Creativity and common sense.

P
a
g
e
1
4


2. Give a definition of marketing research and illustrate it with an example.

Define marketing research as the systematic and objective identification,
collection, analysis, and dissemination of information for the purpose of
assisting management in decision making related to the identification and
solution of problems (and opportunities) in marketing. We also list the
AMAs definition of marketing research, which is worded slightly
differently from the definition offered in the text.

1. I dentification: involves defining the marketing research problem (or
opportunity) and determining the information that is needed to address
it.
2. Collection: data must be obtained from relevant sources.
3. Analysis: data are analyzed, interpreted, and inferences are drawn.
4. Dissemination of information: the findings, implications, and
recommendations are provided in a format that makes this information
actionable and directly useful as an input into decision making.

Any of the examples in the overview may be expanded on and used to
illustrate the components of the definition.


P
a
g
e
1
5


3. Explain and illustrate the classifications of marketing research.

1. Problem identification research. The goal is to identify existing or
potential problems not apparent on the surface. Examples include
market potential, market share, market characteristics, sales analysis,
short-range forecasting, long-range forecasting, and business trends
research.
2. Problem solution research. The goal is to solve specific marketing
problems such as segmentation, product, pricing promotion, and
distribution research.




4. Describe the rationale for the steps involved in the marketing research
process.

Emphasize the importance of these steps since the entire research project
will be conducted within this framework. Each step will be discussed more
fully throughout the course and this is simply an introduction to the general
research process.

P
a
g
e
1
6


1. Problem definition: defining the marketing research problem to be
addressed is the most important step because all other steps will be
based on this definition.
2. Developing an approach to the problem: development of a broad
specification of how the problem will be addressed allows the
researcher to break the problem into salient issues and manageable
pieces.
3. Research design formulation: a framework for conducting the
marketing research project that specifies the procedures necessary for
obtaining the required information. It details the statistical
methodology needed to solve the problem and thus the data
requirements needed from data collection.
4. Fieldwork or data collection: a field force (personal interviewing,
phone, mail, or electronic surveys) gathers project data. While
seemingly trivial in nature, to obtain meaningful results field workers
must be accurate and thorough in data collection.
5. Data preparation and analysis: the editing, coding, transcription, and
verification of data allow researchers to derive meaning from the data.
6. Report preparation and presentation: the findings are communicated
to the client. The report should address the specific research questions
identified in the problem definition, describe the approach, the
research design, data collection and the data analysis procedures
adopted, and present the results and the major findings.

P
a
g
e
1
7


5. Describe the nature of marketing research and illustrate its components
with examples.

Emphasize that marketing research provides the information for decision
makers at each step of the marketing decision process. It is the goal of
marketing research to provide relevant, accurate, reliable, valid, and current
information to management in order to facilitate managerial decisions. Each
of these characteristics can be defined for students as:
o Information that is relevant and addresses the problem or issue being
investigated.
o Information that is accurate, correct, and precise.
o Information that is reliable and originates from competent,
trustworthy sources.
o Information that is valid and applicable to the problem at hand.
o Information that is current, timely and up-to-date for both the industry
and issue under consideration.

6. Discuss the decision to conduct marketing research.
The decision should be guided by a number of considerations including the costs
versus the benefits, the resources available to conduct the research, the resources
available to implement the research findings, and managements attitude toward
research. Marketing research should be undertaken when the expected value of
information it generates exceeds the costs of conducting the marketing research
project. In general, the more important the decision confronting management and

P
a
g
e
1
8

the greater the uncertainty or risk facing them, the greater the value of information
obtained.


7. Discuss the importance of product moment correlation in regression
analysis.

Begin by defining the product moment correlation. From a sample of n
observations of two random variables X and Y, the product moment
correlation is given as

r =
where COV = covariance of X with Y; s = standard deviation.

Also known as the correlation coefficient, r is used as an index to determine
if a linear relationship exists between X and Y. If the variables X and Y are
metric and their distributions have some shape, r is an estimate of R, the
product moment correlation of the population.

8. Discuss the purpose of regression analysis.

COV
xy
S
x
S
y

P
a
g
e
1
9

Introduce regression as a powerful technique for analyzing the association
between a metric dependent variable and one or more independent variables.
It can be used to:
(i) detect the existence of a significant relationship.
(ii) gauge the strength of the relationship.
(iii) determine a mathematical model of the relationship.
(iv) predict the values of the dependent variable.
(v) determine the contributions of a specific variable by controlling other
independent variables.

However, beyond explaining the nature and degree of association between
the variables, regression does not imply or assume causality.
Introduce the general multiple regression model and its assumptions.

Start by mentioning that multiple regression involves a single dependent
variable and two or more independent variables. The assumptions made in
bivariate regression are true for multiple regression also. The general form
of the regression model is:

Y =
o
+
1
X
1
+
2
X
2
+
3
X
3
+ ........ +
k
X
k
+ e

| | | | |

P
a
g
e
2
0

which is estimated by

= a + b
1
X
1
+ b
2
X
2
+ b
3
X
3
+ ........ + b
k
X
k

where the b
i
s represent the partial regression coefficients.


9. Discuss the procedure for conducting n-way analysis of variance.

The procedure for conducting n-way ANOVA is similar to the procedure for
conducting one-way ANOVA, except that there are at least two factors to
account for the total variation. This enables us to examine the interactions
between the factors. If x
1
and x
2
are two factors having c
1
and c
2
categories,
the total variation in this case is given by:

1 2 1 2
total , error y x x x x
SS SS SS SS SS SS = = + + +


The strength of the joint effect of two factors (two main effects plus the
interaction effect) is called the overall effect and is measured by:

Y

P
a
g
e
2
1

( )
1 2 1 2
2 2
multiple multiple
x x x x y
R SS SS SS SS q = = + +


The significance of the overall effect is measured by an F-test, where

1 2 1 2
1 2 1 2
, ,
error
, ,
error
x x x x n
d
x x x x
ss df
F
SS df
MS
MS
=
=



where df
n
= degree of freedom for numerator = (c
1
c
2
1)
df
d
= degree of freedom for denominator = (N c
1
c
2
)
MS = mean square.

The significance of interaction effect is measured by:

1 2
,
error
x x n
d
SS df
F
SS df
= where df
n
= (c
1
1)(c
2
1)
df
d
= N c
1
c
2



P
a
g
e
2
2

The significance of the effect of each factor is given by (shown for x
1
):

1
error
n
SSx df
F
SS
=

Interpretation of results is similar to the one-way ANOVA, but each
independent variable must be examined separately. In addition, the
interaction effect must also be examined. If it is found to be significant, then
the effect of x
1
depends on the level of x
2
and vice versa. This analysis is
based on the assumption that the design was balanced or orthogonal.




10. Describe the statistics used to assess the significance and strength in cross-
tabulation.
The statistical significance of the observed association is measured by the
chi-square statistic. The strength of the observed association is measured by
the phi-correlation coefficient, the contingency coefficient, Cramers V, and
the Lambda coefficient.

1. Chi-Square

P
a
g
e
2
3

Let there be a random sample of n observations and k be the number
of categories. Then the expected frequency for each cell can be
calculated by using a simple formula:

r c
e
n n
f
n
=

where
r
n = total number in the row

c
n = total number in the column
n = total sample size
Let,
o
f = observed frequency
Then the value of
2
_ is calculated as:

( )
2
2
all cells
o e
e
f f
f
_

=



To determine whether a systematic association exists, the probability
of obtaining a value of chi-square as large or larger than the one
calculated from the cross-tabulation is estimated. The chi-square
statistic can also be used in goodness-of-fit tests to determine whether
certain models fit the observed data.

P
a
g
e
2
4


However, the chi-square statistic should only be estimated on counts
of data and not on percentages. It should also be noted that the
observations have to be drawn independently while carrying out the
test. The test should not be conducted if the theoretical or expected
frequency in any of the cells is less than five. In the case of a 2 2
table and when the number of observations in any cell is less than 10,
a correction factor is generally applied.


Questions on Marketing Metrics

1. What do you mean by Share of Wallet/ Share of Requirements?
Ans.) The percentage of customers needs in a category that are served by a given
brand or product.
A given brands share of purchases in its category, measured solely among
customers who have already purchased that brand. Share of requirements is
calculated solely among buyers of a specific brand. Within this group, it represents
the percentage of purchases within the relevant category, accounted for by the
brand in question.





P
a
g
e
2
5

2. How will you measure the usage of a brand?
Ans.) The ratio that compares the average consumption of products in a category
by customers of a given brand with the average consumption of products in that
category by all customers for the category. The usage index can be calculated on
the basis of unit or currency inputs.


3. How will you calculate Net Promoter Score?
Ans.) The Net Promoter Score (NPS) is created by subtracting the percentage of
detractors among current customers from the percentage of promoters among
current customers.
Net Promoter Score (I) = Percentage of Promoters (%) Percentage of
Detractors (%)
For example if a survey of a companys customers reports that there were 20%
promoters, 70% passives, and 10% detractors, the company would have a Net
Promoter Score of 2010 =10.

4. What do you mean by All Commodity Volume (ACV)?
Ans. The percentage of sales in all categories that are generated by the stores that
stock a given brand (again, at least one SKU of that brand).


P
a
g
e
2
6


5. How will you measure Compound Annual Growth Rate (CAGR)?
Ans.) The CAGR is a constant year-on-year growth rate applied over a period of
time. Given starting and ending values, and the length of the period involved, it can
be calculated as follows:


6. What do you mean by Fair Share Draw?
Ans.) The assumption that a new product will capture sales (in unit or dollar
terms) from existing products in direct proportion to the market shares held by
those existing products.

7. How will you measure Brand Equity Index (Moran)?
Ans.) Marketing executive Bill Moran has derived an index of brand equity as the
product of three factors: Effective Market Share, Relative Price, and Durability.
Brand Equity Index (I) = Effective Market Share (%) * Relative Price
(I) * Durability (%)
Effective Market Share is a weighted average. It represents the sum of a
brands market shares in all segments in which it competes, weighted by
each segments proportion of that brands total sales.
Relative Price is a ratio. It represents the price of goods sold under a given
brand, divided by the average price of comparable goods in the market.
Durability is a measure of customer retention or loyalty. It represents the
percentage of a brands customers who will continue to buy goods under that
brand in the following year.


P
a
g
e
2
7

8. How will you measure Customer Lifetime Value?
Ans.) The present value of the future cash flows attributed to the customer
relationship.


9. How will you measure Prospect Lifetime Value?
Ans.) Prospect lifetime value is the expected value of a prospect. It is the
value expected from the prospect minus the cost of prospecting. The value
expected from the prospect is the expected fraction of prospects who will
make a purchase times the sum of the average margin the firm makes on the
initial purchase and the CLV of the newly acquired customer.
Only if prospect lifetime value is positive should the firm proceed with the
planned acquisition spending.
Purpose: To account for the lifetime value of a newly acquired customer
(CLV) when making prospecting decisions.
The expected prospect lifetime value (PLV) is the value expected from each
prospect minus the cost of prospecting.

If PLV is positive, the acquisition spending is a wise investment. If PLV is
negative, the acquisition spending should not be made.

10. What is conjoint analysis?

P
a
g
e
2
8

Ans.)
- Marketing managers frequently want to know what utility a particular
product feature or service feature will have for a consumer.
- Conjoint analysis is a multivariate technique that captures the exact levels of
utility that an individual customer puts on various attributes of the product
offering. It enables a direct comparison between say, the utility of a price
level of Rs. 400 versus Rs.500, a delivery period of 1 week versus 2 weeks,
or an after sales response of 24 hours versus 48 hours.
- Once we know utility levels for every attribute (and at every level), we can
combine these to find the best combination of attributes that gives him the
highest utility, the second best combination that gives the second highest
utility, and so on.
- This information can be used to design a product or service offering.
- If this is done across a sample of customers say, segment-wise, it can also be
used to predict market-share, and the response of customers to changes in
the competitive strategy through changes in the marketing elements.


Rural Marketing

1. Why Rural Marketing is crucial in todays context?
Ans.: With the ever rising consumption levels (in some cases like motorcycles,
sales have trebled in the last decade), it has become essential to look at the rural
consumer anew & afresh.
2. Whats the essential difference between marketing for urban &
marketing for rural consumers?
Ans.: Consumption levels in the urban scenario are driven by infrastructure,
retail environment, mobility & peer pressure. In the rural scenario, consumption
is an outcome of historical factors, land holdings & caste based associations.

P
a
g
e
2
9

3. What are the common constraints for marketers in rural India?
Ans.: Marketing in the urban sector thrives on consumption, which in turn
depends on retail environment, a healthy market for services & disposable
incomes. All these factors are virtually absent from the rural sector.
4. Do consumers in rural India have similar classification to urban
consumers?
Ans.: Urban India has a top creamy layer of about 5%, followed by a vast
salaried middle class. In rural India, we find a top creamy layer of about 5%.
However, there is no salaried middle class. Instead there is the repatriation
class people who own land as well as receive money from sons, daughters &
relatives who work in urban areas. A lot of people are poor as well.
5. Are rural markets fragmented?
Ans.: Yes. Rural market as a whole presents a scenario of islands of
prosperity. There are states like Punjab & Gujarat which are very rich & some
like Bihar, very poor. Even within Punjab, there would be a lot of variance,
depending on the fertility of soil, roads, electricity, access to water & mandis.
6. What are the variables that a marketer must look at in rural India?
Ans.: The crucial factors that would be of great interest to a marketer would be
the road network, proximity to a big city, availability of electricity & the pattern
of crops. There are very rich pockets of rural consumers in the areas that grow
cash crops.
7. What is the penetration of organised retail in rural India?
Ans.: As per Pradeep Kashyap, the rural marketing guru, only 10,000 out of
about 600,000 villages in India have access to organised retail services.
8. What is the penetration level of typical FMCG products in rural India?
Ans.: Cigarettes have a penetration level of 62%. Packaged tea is at 52%. Toilet
soap is at 47%. And washing powder stands at 55%.
9. What is the rural retail spread?

P
a
g
e
3
0

Ans.: Rural retailing in India accounts for Rs. 1.9 lac crores (Rs. 1.9 trillion)
with about 7.8 million retail shops.
10. What media can be used to promote brands in rural India?
Ans.: Video Vans & Folk Media like theatre, songs, dances, puppet shows,
magic shows are popular means of communication. The challenge however lies
in the variety of languages & lingos spoken in rural India.

Consumer Behaviour
1. What is perception? How is it important?
Ans.: Perception can be understood as a short cut to decision making based on
the information available at that time. The consumer perceives a companys
offering as a brand in his/ her mind.
2. What are beliefs? How is understanding of beliefs important?
Ans.: For a person, beliefs are the background to his behaviour. In India, we
have an abiding belief in family values, education, religious festivals & gold.
3. What is post purchase dissonance?
Ans.: It is the disappointment/ dissatisfaction a consumer may experience if he
believes that the product/ service did not come up to the expectations, or there
was a price-delivery mismatch.
4. What is meant by Western yet Indian?
Ans.: The concept has been developed to explain the fact that we Indians have a
huge fascination with the West because of their wealth & progress levels; yet
we remain proud of our own culture, language & way of life.
5. Is it easy to persuade a consumer to buy a different product or service?
Ans.: Yes & No. If it is something that only marginally interests a consumer
(toothpaste), it may be relatively easy. But if its something that he/ she values

P
a
g
e
3
1

& trusts (school, college, etc.) then it is very difficult to persuade him/ her to
look elsewhere.
6. What is culture? What is sub culture or regionalism?
Ans.: Culture is what we all collectively value. In India, we are all culturally
driven towards our families & communities (neighbourhood, region, caste,
religion). Sub culture is a mix of local language, food habits, communities, etc.
It is the growing regionalism, aided by television & cinema which makes Indian
market a fragmented market.
7. Is the consumer decision making rational or emotional?
Ans.: It is a mix of both. For example, the belief that people have in Maruti as a
car is emotional (Its the best car for Indian conditions), but the specific
model they select may be based on reasons like how much money to spend,
mileage, etc.
8. What is attitude?
Ans.: Attitude is the way we look at things or behave. There is a certain
consistency to our attitude. It may be positive or negative. For example, our
attitude towards film stars & cricketers is mostly positive, but our attitude
towards politicians is irrationally negative.
9. What is peer group influence?
Ans.: Peer group is the imaginary or real group of which we consider ourselves
to be a part (Cool dude, corporate world, idealistic teachers, etc.). It
moulds our behaviour in accordance with what the people in the group are
supposedly doing. For example, I am in the corporate world, so I must wear a
silk tie.
10. What is gender difference?
Ans.: Gender difference is the different way in which Men & Women perceive
or react to something. For example, a woman, even while holding a job, may
think of herself primarily responsible for household work. Women are also
thought to be more emotional than men.

P
a
g
e
3
2

Brands & Integrated Marketing Communications

1. Why are brands important?
Ans.: Brands are the sum total of a companys marketing effort, as perceived or
thought of by a consumer. When a consumer decides to buy a Brand, he is
expressing his approval about one or combination of factors (company
credibility, product quality, packaging, availability, etc.) behind the Brand.
2. Products are realBrands are virtual. What does it mean?
Ans.: In reality, Brand is not a physical entity. You cant touch or taste it. But it
is something that draws a mental/ emotional response (My money is secure in
State Bank of India, Nokia is a highly dependable phone) even before we
consume the product.


3. How are brands positioned?
Ans.: The concept of positioning puts the responsibility of understanding a
consumers mindset on the company. Therefore, a company moves away from
saying The consumer will love our product to Does he really need our
product? If it is so, why? Positioning means finding an empty slot in the
consumers mind for the companys brand to fit in.
4. What is branding?
Ans.: Branding starts with the process to figure out how to position your brand.
Then comes the part of Brand elements (Name, logo, colours, advertising style,
etc.), followed by availability & pricing.
5. What is Brand equity?
Ans.: It is the strength of a brand; the sum total of its value to a consumer. The
challenge lies in understanding the source of a brands value (also called

P
a
g
e
3
3

Brand essence), because it could be anything from the country the Brand comes
from to its manufacturing or service process.
6. What is Integrated Marketing Communications (IMC)?
Ans.: The concept of IMC is about combining the various communication
options available to create a recall for your Brand. The options are endless.
Mass media advertising is often a popular choice, but lately there are options
like magic shows or mall promotions to reach out to consumers.
7. Does advertising have a direct correlation to sales?
Ans.: No. Advertising can merely introduce or inform a consumer about a brand
in the best possible way. However, the ultimate decision to buy can depend on
so many variables like trust, pricing, availability, etc.
8. How do brand promotions compare with advertising?
Ans.: A promotion by definition is short term. You could be promoting your
brand in a corner of city or marketplace. So it is local as well. If designed well,
it can create a short term brand recall at a much lower cost. But through
advertising, you can reach out to millions of consumers at once.
9. What is new Public Relations?
Ans.: The old way of PR is to cultivate a relationship with journalists so your
company is favourably reported in the media. In the new world, with web tools
like youtube & facebook, consumers may directly download or obtain or pass
on information about companies, products & leaders they are interested in. So
the challenge for companies is to groom their leaders for a public role or create
events consumers may directly be interested in.
10. What are some of the new & exciting happenings in events & event
promotions?
Ans.: Flash mob is one exciting development where the consumer is caught
unawares & hence is more likely to react, rather than ignore. The other is to
create sponsored content with TV channels. Similarly, there can be bold

P
a
g
e
3
4

announcements like a company willing to sponsor village level sport to enable
an Olympic level champion to possibly emerge, that make headlines.


Retail Marketing
1) What do you mean by retailing?
- Retailing encompasses the business activities involved in selling goods and
services to final consumers for their personal, family or household use.
- Retailers attempt to satisfy consumer needs by having the right merchandise,
at the right price, at the right place and at the right time.

Characteristics of Retailing
1. There is direct end-user interaction in retailing.
2. It is the only point in the value chain to provide a platform for promotions.
3. Sales at the retail level are generally in smaller unit sizes.
4. Location is a critical factor in retailing.
5. In most retail businesses, services are as important as core products.
6. There are a large number of retail units compared to other units of the value
chain.
7. Final consumers make many unplanned or impulse purchases.
8. Retail customers usually visit a store, even though mail, phone and web
sales have increased.

Functions of Retailing
1. Arranging assortments (Offering an assortment enables their customers to
choose from a wide selection of brands, designs, sizes, colours and prices in
one location).
2. Breaking Bulk / Quantity
3. Holding Stock (to ensure regular availability of their offering)
4. Extending Additional Services (credit, home delivery, Product guarantees,
after-sales-service, dealing with customer complaint, information regarding
new products to their customers etc.)
5. Channel of communication (Upward & downward)

P
a
g
e
3
5

6. Transport and Advertising functions



2) What is open to buy?
Most retailers establish a maximum level of inventory they can afford to have on
hand. The Open to Buy is the amount of merchandise the retailer can still buy
before reaching that ceiling.

3) What is outsourcing?
Contracting with outside people or companies to provide services that were
previously performed in-house by employees.

4) What is private label?
Brands owned by a retailer or retail group rather than by a manufacturer.

5) What do you mean by trade area?
Geographic area from which a store or shopping center will obtain most of its
customers.


6) What is merchandising?
Activities involved in acquiring particular goods and/or services and making
them available at the places, times, and prices and in the quantity that enable
a retailer to reach its goals
Merchandising can be termed as the planning, buying, and the selling of
merchandise.
The planning involved in marketing the right merchandise at the right place
at the right time in the right quantities at the right price.


7) What is slotting fee?

P
a
g
e
3
6

The price retail chains demand to stock an item they do not carry. The amount of
the slotting fee may vary with the space, display, and promotional support to be
allotted to the new item.

8) What is cross merchandising?
Using different lines of goods to help sell each other, for example by displaying
them together.

9) What is consignment merchandise?
Merchandise that is placed in a store but remains the property of the supplier and is
paid for by the retailer only when it is sold. Consignment merchandise usually may
be returned to the supplier whenever the retailer wishes.

10) What do you mean by off-price retailing?
Stores offering well-known brands of merchandise at substantially lower prices
compared to conventional stores handling the same brands.



CRM
1. What is CRM?

CRM is the acronym for Customer Relationship Management, a strategy for
managing interactions with clients and prospects. CRMs use technology to
organize and automate business processes. The ultimate goal of implementing a
CRM is to find new leads, transfer prospects into clients and retain existing clients.
A combination of business strategy, software and business processes which, if
implemented correctly, will bring significant, tangible benefits to your company


P
a
g
e
3
7

2. What are the benefits of CRM?
CRM systems bring significant benefits to businesses. They will help you gain
more customers and keep existing customers coming back. They'll help with your
lead management, your marketing, improve your reports and forecasting and
greatly enhance customer service. As well as strategic level benefits, modern CRM
systems bring benefits on a day-to-day level, increasing staff efficiency and
effectiveness. In summary, a new CRM system will provide major benefits
throughout a company: if it is deployed correctly.
3. What department should be responsible for CRM?
The simple answer is everyone. What we mean is try to spread project
responsibilities across all departments that will have a hand in the success of the
solution. While it is usually necessary to appoint a project leader, the rest of the
team should be comprised of all other departments. This will ensure consistent
buy-in across the organization and enforce team attitudes and ownership. This will
avoid problems associated with lack of employee utilization.
4. What is the CRM Ecosystem?

The CRM ecosystem is the integration of the back office and front office through
the exchange of data, systems, and business processes. The front office provides
complete customer profiles to all customer facing persons. Business Intelligences
systems (data warehouses, data marts and data mining/analytic techniques) capture
information from all customer touch points. Workflow and business rules, along
with information from back-office systems such as inventory control, transmit
business intelligence to front-office personnel to enable them to better service their
customers. Finally, performance metrics enable the company to evaluate their
efforts.

5. Why are so many companies and organizations developing CRM systems
today?
A straightforward answer to why CRM systems are being used is that they
can enhance productivity across the entire range of key marketing functions:
A. Identifying prospects

P
a
g
e
3
8

B. Acquiring customers
C. Developing customers
D. Cross-selling
E. Up-selling
F. Managing migration
G. Servicing
H. Retaining
I. Increasing loyalty
J. Winning back defectors

6. What is Touch Point?

A touch point is any point of contact that a customer or prospect has with the
company. It can be simply an inquiry over the phone, in-person or via e-mail,
or a purchase transaction. It can be in a store, over the Internet, over the
telephone, at an ATM, or kiosk at a trade-show.

7. What are the major challenges that companies encounter when trying to
implement a CRM system?

The real barriers to a smooth CRM introduction appear to be corporate
culture, employee attitudes, organizational structure, and the lack of a
strategic plan for the CRM system as opposed to technology, the need to
replace or integrate existing systems, or the amount of time involved in
establish the new CRM system. In addition, CRM systems interface with a
larger group of stakeholders with more diverse needs than most other
business systems, making it more difficult for organizations to effect the
many cultural, psychological, and structural changes needed for successful
CRM implementation. Executives worry about payback, salespeople are
skeptical of having to learn yet another system, managers worry about
managing their business during the implementation phase, and customers
worry about privacy.

8. What is computer telephony?

P
a
g
e
3
9

Computer telephony describes the hardware and computer software
technologies that enable communication of voice and information via the
phone.
9. What is a data warehouse?
A data warehouse is a data repository or database that contains all
relevant customer and prospect information including respective
marketing information. Pg 206
10. What is a data mart?
A data mart is a data repository or database which is subset of a data
warehouse. It contains less breadth and depth of information than a data
warehouse and is usually accessed by software designed for a specific
business function.


Entrepreneurship and Innovation:

1. What is a corporate entrepreneur?
A corporate entrepreneur is a person with average to above-average
intelligence who has the ability to perceive the big picture. They are not
stagnated by structure and have learned how to manipulate it. They are people
who like to see things happen and they like to make them happen. They are
self-confident and optimistic. They are willing to work hard with an idea, not
needing an authoritative figure over them and are self-motivated.

2. What are the four causes of stress among entrepreneurs? How can an
entrepreneur deal with each of them?

There are four causes of stress: loneliness, immersion in business, people
problems, and need to achieve. To reduce stress, entrepreneurs must define the
cause of the stress. After clarifying the cause of stress, the entrepreneur can

P
a
g
e
4
0

combat excessive stress by acknowledging its existence, developing coping
mechanisms, and probing personal unacknowledged needs.

3. What are two reasons that such a strong desire to develop corporate
entrepreneurs has arisen in recent years?

A highly competitive market has risen. In order for a company to survive, it
must be innovative. Also, many firms that are unwilling to change are losing
their best people to entrepreneurship.

4. What are some of the corporate obstacles that must be overcome to
establish a corporate entrepreneurial environment?

The firm needs to emphasize individual responsibility among its employees. It
must have a system of rewards based on results. It must overcome traditional
management practices that are too rigid.

5. Identify the four key elements on which managers should concentrate so
as to develop a corporate entrepreneurship strategy.

The four key elements are: developing a vision; encouraging innovation;
structuring for an entrepreneurial environment; and developing venture teams.

6. Explain the differences between radical and incremental innovation.

Radical innovation represents inaugural breakthroughs that have been launched.
These innovations take experimentation and vision but are not managed.
Incremental innovation refers to the systematic evolution of a product or service
into a new market. Innovation can be managed.

7. Identify the five specific entrepreneurial climate factors that
organizations need to address in structuring their environment.
Top management support; autonomy/work discretion; rewards/reinforcement;
time availability; and organizational boundaries


P
a
g
e
4
1

8. What are roles of middle managers in corporate entrepreneurship?

Middle managers endorse, refine, and shepherd entrepreneurial opportunities
and identify, acquire, and deploy resources needed to pursue those
opportunities. The entrepreneurial actions expected of middle managers are
framed around the need for this group to propose and interpret entrepreneurial
opportunities that might create new business for the firm or increase the firms
competitiveness in current business domains

9. Describe opportunity identification for the entrepreneur.

Opportunity identification is the central domain of entrepreneurship. The first
step for any entrepreneur is the identification of a good idea, but the search
for good ideas is never easy. At the core of entrepreneurship lies the questions
of how, why, and when opportunities for the creation of goods and services
arise in an economy. When the entrepreneur has discovered a good
opportunity, they are well on their way to becoming more serious about starting
a business.

10. What is the difference between an adaptor and an innovator?

An adaptor is one who takes the ideas of another and tries to make them
profitable. There is no creativity involved because the person is not trying to
profit from his own idea, but from that of another. An innovator is one who
looks for an opportunity on which to capitalize. The innovator researches the
need for the idea by finding out the thoughts of the potential customers. An
innovator also thinks very carefully about what exactly the innovation has to be
in order to appeal to the public.
11. In your own words, state what is meant by the term innovation.

Innovation means something new or something that has not been done before.
It could be thought of as a new idea for a certain type of product or service.
Innovation could also be described as an improvement of an existing idea,
which makes the idea profitable.

P
a
g
e
4
2

12. Identify and describe five of the innovation principles.
(1) Be action-oriented. Innovators must be active and searching for new ideas,
opportunities, or sources of innovation.
(2) Make the product, process, or service simple and understandable. People
must readily understand how the innovation works.
(3) Start small. Innovators should start small and then build and develop,
allowing for planned growth and proper expansion in the right manner and
at the right time.
(4) Aim high. Innovators should aim high for success by seeking a niche in
the marketplace.
(5) Work, work, work. It takes work, not genius or mystery, to innovate
successfully.

B2B Marketing
1. What does B2B stand for?
B2B stands for business-to-business. This phrase describes businesses that sell
products and/or services to other businesses rather than to individual consumers.
2. What are the features of business to business marketing?
In B2B marketing, transactions are made between and within value chains. It has a
small numbers of customers who require personalized marketing, including
customized products and prices. Selling processes are complex and lengthy, and
involve lots of players creating a demand decision chain. B2B is also noted for
developing profitable, value-oriented relationships between two companies and
several individuals within them.
3. What do you understand by B2B marketing strategy?
It is a set of programs coupled with the target market opportunities in order to
achieve organizational goals. Shaping this strategy involves three steps target

P
a
g
e
4
3

market choice, setting marketing objectives and building the B2B marketing
program.
4. What do you understand by Commission Merchants?
Commission Merchants represent sellers / manufactures, mostly with bulk
commodities like raw materials, to perform functions like arranging inspection,
transporting, negotiating and selling. They are paid commission on the value of
sales.
5. Why Industrial Demand is called Derived Demand?
Industrial demand is called Derived Demand because Industrial demand is
derived from (or depends on) demand for consumer goods/ services.
6. What are the different types of Business to Business Customers?
Different types of Business to Business Customers are:

(i) Commercial Enterprises, (ii) Government
(iii) Institutional, (iv) Cooperative societies.
7. What are the various types of Industrial Products/Services?
Industrial Products/Services are classified into:
(i) Materials & Parts, (ii) Capital Items, (iii) Suppliers & Services.
8. What is Organizational Buying?
Organizational buying refers to the decision-making process by which formal
organizations establish the need for purchased products and services, and identify,
evaluate, and choose among alternative brands and suppliers.
9. What are the different steps of effective communication of Business to
Business Marketing?
The steps involved are:

P
a
g
e
4
4

(i) Decide communication objectives
(ii) Identify the target audience
(iii) Decide the promotional budget
(iv) Develop the message strategy
(v) Select the media
(vi) Evaluate the promotions results
(vii) Integrate the promotions programme

10. What are the different business promotion tools available for business to
business marketing?
Different business promotion tools are: Trade shows (or exhibitions), sales
contests, promotional novelties (or specialty advertising, or gifts), seminars,
catalogues, promotional letters, demonstration, and entertainment.


FAQs (Finance)

1. What is goodwill? How does it affect net income?
Answer: Goodwill is the value of reputation of a irm in respect of the profits
expected in future over and above the normal rate of return, which other companies
can earn. Over and above the normal rate implies that the firms capability to earn
more profits when compared to other firms because of its good brand name,
locational advantage, good customer relations or possession of a unique patent
right. The impact of goodwill on the net income is that, as good will is amortized

P
a
g
e
4
5

the amount of profits gets reduced. This further reduces the balance of reserves and
surplus amt in the balance sheet.

2. What is working capital?
Answer: A measure of both a company's efficiency and its short-term financial
health. The working capital ratio is calculated as: Working Capital = current asset
Current liabilities.

3. Explain fixed cost, variable cost and marginal cost?
Answer: Fixed cost: A cost that remains constant with the change in volume of
production.

4. What is Variable cost?

Variable Cost: expense that varies in proportion to the quantity of goods sold
or manufactured, such as packaging materials or product components.

5. What is Marginal Cost?

Answer: In economics and finance, marginal cost is the change in total cost that
arises when the quantity produced changes by one unit. That is, it is the cost of
producing one more unit of a good.


6. What is Break Even Point? What does it signify?

Answer: The point at which sales equal costs. The point is located by breakeven
analysis, which determines the volume of sales at which fixed and variable costs
will be covered. All sales over the breakeven point produce profits; any drop in

P
a
g
e
4
6

sales below that point will produce losses.

7. What is authorized share capital?

Answer: The authorised capital of a company (sometimes referred to as
the authorised share capital, registered capital or nominal capital, particularly in
the United States) is the maximum amount of share capital that the company is
authorised by its MOA (Memorandum of Association).

8. Ratios which are useful for Management: Operating Ratio, Debtors
Turnover ratio, Stock turnover Ratio, Solvency ratio and Return on Capital
employed.

9. Ratios useful for Creditors: Current Ratio, Solvency Ratio, Fixed Ratio
and Creditors turnover ratio.


10. Ratio useful for Shareholders : Price-earning ratio, Dividend ratio,
Dividend payout ratio and Return on shareholders fund

Profitability Ratio:
11. Gross Profit % = Gross Profit * 100
Net Sales

12. Net Profit % = Net Profit after tax * 100
Net Sales

13. Operating Expense Ratio = operating Expense * 100
Net Sales

14. Operating Expenses= Admin. Expenses+ selling& dist.expenses+ Financial
expenses

P
a
g
e
4
7


15. Return on Assets = Net Profit * 100
Average Total tangible Assets

16. Return on Equity = Net Profit after interest, tax & pref.dividend *100
Equity share capital
Equity share capital = equity share capital + P/L a/c + reserve & surplus

17. Return on capital employed =
Net profit before interest & tax * 100
or ROI Average Capital employed

Average capital employed = Op. capital employed + closing employed
2
or
Capital Employed= Total asset Current liabilities
18. Return on Shareholders fund = Net Profit after Interest & Tax * 100
Shareholders Funds
Shareholders Fund= Equity share capital +Preference Share capital+ P/L a/c +
Reserve & surplus
Liquidity or Short-Term Solvency ratios:
19. Short-term funds management
Working capital management is important as it signals the firms ability to
meet short term debt obligations.
20. Current Ratio : The ideal benchmark for the current ratio is 2:1. The
acceptable benchmark is 1: 1.Working Capital = Current assets Current
Liabilities
Current Ratio = Current Assets
Current Liabilities
21. Quick Ratio/Acid test Ratio =
Current Assets - ( Inventory + Prepaid expenses)
Current Liabilities

P
a
g
e
4
8

22. Super quick Ratio = Cash in hand + Cash at bank + Mkt. securities
Current liabilities
Activity Ratios:
Efficiency of asset usage. How well assets are used to generate revenues
(income) and its impact on the overall profitability of the business. It is
calculated by:-
23. Asset Turnover Ratio : This ratio represents the efficiency of asset usage to
generate sales revenue
24. Working Capital Turnover Ratio = Cost of goods sold(COGS)
Working Capital
25. Working Capital= CA- CL
26. COGS = sales- Gross Profit
27. COGS = Opening stock + (Purchase- purchase return) + Direct Expenses
closing stock
28. Asset Turnover = Net Sale
Average Total Assets
29. Inventory Turnover = Cost of Goods Sold
Average Inventory/stock

30. Average Inventory= Opening Invent. + closing Invent.
2

31. Debtors Turnover Ratio= Net credit sales
Average Debtors

32. Average debtors = Op. Drs+ Cl. Dr. + B/R
2

33. Average Collection Period = Months or days in a year
Debtors Turnover
or Average Dr x months or days in a year
Net credit sales for the year

P
a
g
e
4
9

34. Creditors Turnover Ratio =
Net Credit purchase
Average accounts payable
35. Average accounts payable=
Op. Cr. + Closing Cr. + B/P
2
36. Average Payment Period =
Months or days in a year
Creditors turnover ratio
or
= Average Cr. Or B/P x months or days in a year
Net credit purchase

37. Financial Structure or Capitalisation Ratios/Solvency ratio: Long term
funds management
Measures the riskiness of business in terms of presence of capital portion i.e.
debt or equity.
For example: Debt/Equity
This ratio measures the relationship between debt and equity. This ratio
indicates that debt and equity funding are equal (i.e. there 1 of debt to 1 of
equity) whereas a ratio of 1.5 indicates that there is higher debt gearing in
the business (i.e. there is 1.5 of debt to 1 of equity). This higher debt gearing
is usually interpreted as bringing in more financial risk for the business
particularly if the business has profitability or cash flow problems.

38. Debt/Equity ratio = Debt / Equity or
External equity
Internal Equity
39. Debt/Total Assets ratio = Debt *100
Total Assets
40. Equity ratio = Equity *100
Total Assets
41. Capital gearing ratio= Fixed Interest bearing funds
equity share capital

P
a
g
e
5
0

Fixed interest bearing security = Preference share capital + debentures +
loan &advances
Market Test Ratios:
42. Earnings per share = Net Profit after tax
Number of issued ordinary shares
43. Dividends per share = Dividends
Number of issued ordinary shares
44. Dividend payout ratio =
Dividends per share *100
Earnings per share
45. Price Earnings ratio =
Market price per share
Earnings per share
45. What is the difference between inactive accounts and dormant account?
Dormant accounts are those accounts in which there are transactions in the recent
history (the stipulation may vary according to the company's rules). Inactive
accounts are those accounts in which transactions are being made for long time.
Inactive account means an account had not been in use for a long time and not
going to be used in the future.Dormant accounts are those which are presently not
in use but it is likely that it may be used in the near future.
46. What is loan of hypothecation?
When a person pledges a mortgage as collateral for a loan, it refers to the right that
a banker has to liquidate goods if you fail to service a loan. The term also applies
to securities in a margin account used as collateral for money loaned from a

P
a
g
e
5
1

brokerage.

47. What is Treasury Bill?
Answer: Treasury Bills are money market instruments to finance the short term
requirements of the Government of India. These are discounted securities and thus
are issued at a discount to face value.

48. Give the details of a Cash Flow Statement?
Answer. Start with net income, go line by line through major adjustments
(depreciation, changes in working capital and deferred taxes) to arrive at cash
flows from operating activities. Mention capital expenditures, asset sales,
purchase of intangible assets, and purchase/sale of investment securities to
arrive at cash flow from investing activities. Mention repurchase/issuance of
debt and equity and paying out dividends to arrive at cash flow from financing
activities. Adding cash flows from operations, cash flows from investments, and
cash flows from financing gets you to total change of cash. Beginning-of-period
cash balance plus change in cash allows you to arrive at end-of-period cash
balance.
49. What is EBDIT?
Answer: EBITDA stands for earnings before interest, taxes, depreciation, and
amortization. EBITDA came into wide use among private capital firms,
wanting to calculate what they should pay for a business.
50. What is Takeover?
Answer: In business, a takeover is the purchase of one company (the target) by
another (the acquirer, or bidder). In the UK, the term refers to the acquisition of
a public company whose shares are listed on a stock exchange, in contrast to the
acquisition of a private company.

Hostile takeovers are when another company, often using leverage (loans from
banks and investment houses that add power to buy the company in question)
try to buy enough shares in the company to control it. "Hostile" simply means it
is against the wishes of the company



P
a
g
e
5
2

Glossary of Finance

1) Balance of Power: More commonly referred to as BOP, is a proprietary
technical indicator calculated from both a securitys price and volume. The
primary purpose of BOP is to help uncover periods of systematic accumulation
(buying) and systematic distribution (selling).
2) Money Stream: An indicator to view the price vs. volume of a stock. If money
stream is moving upwards at a greater angle than the price, it shows the stock in a
bullish stage.
3) Beta: Beta is a measure of the volatility (aka instability or risk) of a stock
offering relative to the overall market average. A Beta of less than one specifies
lower risk than the market; a Beta greater than one denotes a higher risk than the
overall stock market.
4) Common Stocks: The basic, most common form of equity ownership in a
corporation. Each share of common stock represents an equity interest in the
company, a right to vote on important matters, and the right to an equitable share
of the companys success in the form of dividends or capital appreciation.
Preferred stock supersedes common stock in first right to corporate dividends.
5) Dividend: Dividends are payments made by a corporation to its shareholder
members. It is the portion of corporate profits paid out to stockholders. When a
corporation earns a profit or surplus, that money can be put to two uses: it can
either be re-invested in the business (called retained earnings), or it can be paid to
the shareholders as a dividend. Many corporations retain a portion of their earnings
and pay the remainder as a dividend.
6) Asset allocation: is the process of distributing your investment assets in a
manner that is consistent with your investment goals among different classes of
investments.
7) A stock screen: is a software tool, usually Internet-based, that allows investors
to define a set of criteria for his or her investment in a stock. The screen then finds
all stocks that match the criteria. Also Known As: Stock Screening. Examples: A
stock screen might include the following criteria: P/E of <25; annual EPS growth
of 10% for past 5 years; in Health Care sector.
8) The Securities and Exchange Board of India (SEBI): is the chief regulatory body
over the stock markets and companies that are publicly traded.

P
a
g
e
5
3

9) P/E: The P/E ratio (price-to-earnings ratio) of a stock (also called its earnings
multiple, or simply multiple, P/E, or PE) is a measure of the price paid for
a share relative to the annual income or profit earned by the firm per share. A
higher P/E ratio means that investors are paying more for each unit of income. It is
a valuation ratio included in other financial ratios. The reciprocal of the P/E ratio is
known as the earnings yield. Stock having a P/E less than 30 are said to be good
investments.
10) Annual General Meeting - A meeting of the shareholders that must be held
every calendar year to enable them to view the records of the company, elect
directors and vote on matters integral to the running of the company.
11) Articles of Association - Documents required by Corporations Law detailing
the rules for the internal management of the company.
12) Bear - A person who expects prices to fall and sells securities hoping to make a
profit by subsequently repurchasing at a lower price.
13) Bid - The price at which someone is prepared to buy/sell shares.
14) Blue Chip Shares - Shares of a company known for its ability to make
profits/pay a dividend in good times or in bad. Yield is often proportionately low.
15) Debenture/Bond - Document recording a loan made to a government or semi-
government body or private for a fixed period of time at a fixed rate of interest.
16) Bonus Issue - The issue of bonus or free shares to existing shareholders,
usually in a predetermined ratio, eg. one bonus share for every three shares held.
17) Boom Market - A market in which buying demand greatly exceeds selling
pressure. In these circumstances, prices rise.
18) Brokerage - Charges made by a broker for acting as a agent in the buying and
selling of shares.
19) Bull - A person who buys securities in the expectation that price will rise and
so give him an opportunity to resell at a profit.
20) Call Option - an option giving the taker the right, but not the obligation, to buy
the underlying shares at a specified price on or before a specified date.
21) Diversification - Spreading an investment across a number of assets will
eliminate some, but not all of the risk.
22) Derivatives:- is a financial instrument or more simply, an agreement between
two parties that has a value determined by the price of something else(called
underlying). It is a financial contract with a value linked to the expected future
price movements of the asset it is linked to - such as a share or a currency. There

P
a
g
e
5
4

are many kinds of derivatives, with the most notable being swaps, futures, and
options people or two parties - that has a value determined by the price of
something else (called the underlying)
23) Forward: - is a contract made today for delivery of an asset at a prespecified
time in the future at a price agreed upon today. The buyer of a forward contract
agrees to take delivery of an underlying asset at a future time, T, at a price agreed
upon today. No money changes hands until time T. The seller agrees to deliver the
underlying asset at a future time, T, at a price agreed upon today. Again, no money
changes hands until time T. A forward contract, therefore, simply amounts to
setting a price today for a trade that will occur in the future
24) Future:- is a standardized contract between two parties to buy or sell a
specified asset of standardized quantity and quality at a specified future date at a
price agreed today (the futures price). The contracts are traded on a futures
exchange
25) Dividend: - Distribution of part of a company's net profit to share holders as a
reward for investing in the company. Usually expressed as a percentage of par
value or as Rs per share.
26) Depositories: - A depository is an organisation which holds securities (like
shares, debentures, bonds, government securities, mutual fund units etc.) of
investors in electronic form at the request of the investors through a registered
Depository Participant. It also provides services related to transactions in
securities. At present two Depositories viz. National Securities Depository Limited
(NSDL) and Central Depository Services (India) Limited (CDSL) are registered
with SEBI.
27) Earnings Per Share (E.P.S.) - Company's net profit divided by the total number
of shares in the company. Expressed as Rs per share.
28) ESOP:- Employee stock option plan. Mostly offered to management as part of
their compensation /incentive salary package.
29) Interim Dividend - A dividend paid during the year and not at the end. Most
profitable companies pay dividends every half year.
30) Liquidator - A person appointed to take charge of a company when it is wound
up.
31) Market Capitalization: The product of the number of the companys
outstanding ordinary shares and the market price of each share.

P
a
g
e
5
5

32) Large-Capitalization: A term used by the investment community to refer to
companies with a market capitalization value of more than $10 billion. Large cap
is an abbreviation of the term "large market capitalization".
33) Mid-Capitalization: firms generally ranging from $2 billion to $10 billion in
market capitalization
34) Small-Capitalization: company with a market capitalization of between $300
million and $2 billion
35) Micro cap: market capitalization of less than $ 300 million.
36) Placement: An allotment of shares, debentures, etc. made directly from the
company to selected investors.
37) Portfolio: Investors holding of securities of various types. The wise investment
policy is to build up a balanced portfolio according to personal requirements.
38) Preference Shares:- Rank above ordinary shares for claims on assets, earnings
and dividends but rank below creditors and debenture holders. These shares
usually have a fixed dividend rate.
39) Prospectus: Document issued by a company setting out the terms of its public
equity issue or debt raising.
40) Stockbroker: A Stock Exchange Member who buys and sells stocks, shares and
securities for clients.
41) Systematic Risk: A risk that influences a large number of assets. Also known
as market risk.
42) Trust: Investments which involve pooling investors' money with experts
managing the investment for the individuals. Trusts usually concentrate on one
area of investment. The three most common are Equity, Property and Cash
Management.
43) Underwriter: one who arranges a new issue of securities and agrees to purchase
any unsold securities thereby guaranteeing full subscription.
44) Unsystematic Risk: A risk that affects at most small number of assets. Also
known as unique or asset-specific risks.
45) Warrants: Call option issued by a company
46)STT:Security Transaction Tax
47) ELSS: Equity linked saving schemes
Governing body of life insurance in India: - IRDA (Insurance regulatory and
development authority)

P
a
g
e
5
6

48) Negotiable Instruments: Negotiable instruments are written orders or
unconditional promises to pay a fixed sum of money on demand or at a certain
time. Promissory notes, bills of exchange, checks, drafts, and certificates of deposit
are all examples of negotiable instruments. Negotiable instruments may be
transferred from one person to another, who is known as a holder in due course.
49) Retained earnings: Net profits kept within a business in the Owners' Equity
account after stock dividends are paid.
50) Net Asset Value: The underlying value of a share of stock in a particular
mutual fund.
51) Close-ended schemes: Such funds have a fixed maturity period and are open
for subscription only for a specified period. After the expiry of this period,
investors can buy or sell the units on the stock exchanges where such funds are
listed. Some funds also have the option of periodic repurchase, whereby investors
can sell back their units to the fund at NAV related prices.
52) Balanced schemes: Such funds have a balanced portfolio and invest in equity
and preference shares in addition to fixed income securities. The aim of such funds
is to provide both income and capital appreciation over a long-term.
53) Money Market: Market in which short-term securities are bought and sold.
54) Mutual Fund: A company that invests in and professionally manages a
diversified portfolio of securities and sells shares of the portfolio to investors.
55) Offer for Sale: An offer to the public by, or on behalf of, the holders of
securities already in issue.
56) Offer for Subscription: The offer of new securities to the public by the issuer or
by someone on behalf of the issuer.
57) Open-end (Mutual) Fund: There is no limit to the number of shares the fund
can issue. The fund issues new shares of stock and fills the purchase order with
those new shares. Investors buy their shares from, and sell them back to, the
mutual fund itself. The share prices are determined by their net asset value.
58) Open Offer: An offer to current holders of securities to subscribe for securities
whether or not in proportion to their existing holdings.
59) Option: A security that gives the holder the right to buy or sell a certain
amount of an underlying financial asset at a specified price for a specified period
of time.
60) Oversubscribed: When an Initial Public Offering has more applications than
actual shares available. Investors will often apply for more shares than required in

P
a
g
e
5
7

anticipation of only receiving a fraction of the requested number. Investors and
underwriters will often look to see if an IPO is oversubscribed as an indication of
the publics perception of the business potential of the IPO company.
61) Par Bond: A bond selling at par (i.e. at its face value).
62) Par Value: The face value of a security.
63) Statutory Liquidity Ratio or SLR refers to the amount that all banks require to
maintain in cash or in the form of Gold or approved securities. Here by approved
securities we mean, bond and shares of different companies.
64) Repo (Repurchase) Rate: Repo rate is the rate at which banks borrow funds
from the RBI to meet the gap between the demand they are facing for money
(loans) and how much they have on hand to lend. If the RBI wants to make it more
expensive for the banks to borrow money, it increases the repo rate; similarly, if it
wants to make it cheaper for banks to borrow money, it reduces the repo rate
65) Reverse Repo Rate: This is the exact opposite of repo rate. The rate at which
RBI borrows money from the banks (or banks lend money to the RBI) is termed
the reverse repo rate. The RBI uses this tool when it feels there is too much money
floating in the banking system
66) Bank Rate: This is the rate at which RBI lends money to other banks (or
financial institutions .The bank rate signals the central banks long-term outlook
on interest rates. If the bank rate moves up, long-term interest rates also tend to
move up, and vice-versa. Banks make a profit by borrowing at a lower rate and
lending the same funds at a higher rate of interest. If the RBI hikes the bank rate
(this is currently 6 per cent), the interest that a bank pays for borrowing money
(banks borrow money either from each other or from the RBI) increases. It, in turn,
hikes its own lending rates to ensure it continues to make a profit.
67) Call Rate: Call rate is the interest rate paid by the banks for lending and
borrowing for daily fund requirement. Since banks need funds on a daily basis,
they lend to and borrow from other banks according to their daily or short-term
requirements on a regular basis.
68) CRR: Also called the cash reserve ratio, refers to a portion of deposits (as cash)
which banks have to keep/maintain with the RBI. This serves two purposes. It
ensures that a portion of bank deposits is totally risk-free and secondly it enables
that RBI control liquidity in the system, and thereby, inflation by tying their hands
in lending money

P
a
g
e
5
8

69) Actuary: A person trained in mathematics, statistics, and accounting who is
responsible for determining premium rates, reserves, and dividends as well as
conducting various other statistical studies.
70) Indemnity: A principle that says an insured should not collect more from
insurance than the amount of loss.
71) Subrogation: Gives the insurer whatever right against third parties you may
have as a result of the loss for which the insurer paid you.
72) Insurable Interest: If the occurrence of a loss, such as destruction of a house by
fire, will affect you adversely, you have an insurable interest.
73) Insured: In life insurance, the person on whose life a policy is issued; the
subject of insurance. In property and liability insurance, the person to whom, or on
whose behalf, benefits are payable.
74) Peril: The cause of a possible loss.
75) Ratios and its Analysis:- ratio is a mathematical relationship between two
interrelated variables in financial statement. Ratio analysis is a tool used by
individuals to conduct a quantitative analysis of information in a company's
financial statements. Following ratios are used to judge the financial health of
company- Profitability ratio, liquidity ratio, leverage ratio and activity or turnover
ratio.
76) Net present value: A method for evaluating investment proposals. NPV is
defined as present value of benefits minus present value of costs. For accepting any
project, NPV should be positive.
77) Internal Rate of Return: the rate of discount at which the net present value of
an investment is zero. The value of IRR should be more than the cost of capital of
the firm for the project to be accepted.

International Business
Q1.what is Bill of Lading?
Ans. Bill of Lading is a document that establishes the terms of contract between a
shipper and a transportation company. It serves as a document of title, a contract of
carriage and a receipt for goods
Q2. What do you mean by Break bulk?

P
a
g
e
5
9

Ans. Break bulk is loose, non containerized cargo
Q3. What do you mean by Cash in Advance?
Ans. Payment for goods if the price is paid in full before shipment is made.
Q4.What is Charter party?
Ans. A written contract,usually on a special form between the owner of a vessel
and a "charterer"who rents use of the vessel or a part of its freight space.
Q5.What is CIF (Cost, Insurance and Freight)
Ans. It means that a price includes the various costs, such as transportation and
insurance, needed to get good from one country to another.
Q6 . What do you mean by FOB?
Ans. Free on Board, the seller clears the goods for transport and delivers them on
board the ship.
Q7. What is a Contract?
Ans. A legally binding agreement between two or more persons/organizations to
carry out reciprocal obligations or value
Q8. What do you mean by "Country of origin"?
Ans. The country in which a good was produced.
Q9. What is Currency risk?
Ans. Uncertainty about the future value of a currency
Q10. What do you mean by D/P (Documents against Payment)?
Ans. Term for documentry draft collection instructions requesting the presenting
bank to deliver documents only upon acceptance of the draft by the drawee/
importer

P
a
g
e
6
0

Q11. What is the meaning of Dry Cargo?
Ans. Cargo that is not liquid and normally does not require temperature control
Q12. What do you mean by ETA?
Ans. Estimated time of arrival (of vessel) of a vessel.
Q13.The sense of belonging (group) has got priority in which culture?
Ans. Japan
Q14.The theory of absolute advantage was given by?
Ans. Adam Smith
Q15. Which country attracts the largest FDI inflow?
Ans. China
Q16. What do you mean by Airway bill?
Ans. The receipt issued by an airline or its agent for carriage of goods .
Q17. McDonald's entry strategy for international market is called
Ans. Franchising
Q18.What do you mean by Gray Market?
Ans. Refers to goods that are sold for a price lower than,or through a distributor
different than,that intended by the manufacturer.
Q19.What do you mean INCOTERMS?
Ans. International Commercial Terms like FOB, CIF ect.
Q20. What is Multimodal Transport?

P
a
g
e
6
1

Ans. Shipment of goods by more than one means of transportation.

Q21. What do you mean by Multinational Enterprise?
Ans. A firm, usually a corporation that operates in two or more countries.
Q22.What is the full form of APEDA?
Ans. Agricultural and Processed Food Products Export Development Authority
Q.23 What is MDA (Market Development Assistance)?
Ans. Assistance given to exporters and export promotion organizations for market
exploration and export promotion on cost-sharing basis.
Q24. What do you mean by EPCG (Export Promotion Capital Goods
scheme)?
Ans. The scheme allows for the import of capital goods at concessional rate of duty
subject to an appropriate export obligation accepted by the exporter
Q25. What do you mean by Shipment advice?
Ans. Soon after the shipment has taken place, the shipment advice is sent to the
importer informing him of the details of the shipment.


Supply Chain Management

1. Define PROCESS APPROACH for businesses?

P
a
g
e
6
2

In order to achieve the main objective of the company, the managers should think
the business as a set of processes which integrate to achieve the goal, and not as
individual departments, and should act to achieve the main goal and not individual
goal. Process owners are responsible for ensuring that the business process meets
its objective.
2. What do you understand by the term SILO THINKING?
An information silo is a management system incapable of reciprocal operation with
other, related management systems. A bank's management system, for example, is
considered a silo if it cannot exchange information with other related systems
within its own organization, or with the management systems of its customers,
vendors, or business partners. The expression is typically applied to management
systems where the focus is inward and information communication is vertical.
3. Describe PORTAR VALUE CHAIN?
Porter argues that the ability to perform particular activities and to manage the
linkages between these activities is a source of competitive advantage. Porter
distinguishes between primary activities and support activities. Primary activities
are directly concerned with the creation or delivery of a product or service. They
can be grouped into five main areas: inbound logistics, operations, outbound
logistics, marketing and sales, and service. Each of these primary activities is
linked to support activities which help to improve their effectiveness or efficiency.
There are four main areas of support activities: procurement, technology
development (including R&D), human resource management, and infrastructure
(systems for planning, finance, quality, information management etc.)
4. What is BASIC UTILITY of a product?
An economic term referring to the base satisfaction received from consuming a
good or service.
5. What is augmented AUGMENTED UTILITY of a product?
Augmented means added to or made greater in amount or number or strength. So
augmented utility means having additional features which increases its utility.eg-

P
a
g
e
6
3

need of a shirt is a utility but the shirt should be comfortable is an augmented
utility.
6. How do we identify VALUE ADDING ACTIVITIES in a value chain?
That generates a positive return on the investment of resources and cannot be
eliminated without impairing a process. Activities which are directly contributing
in change of product shape or value in a process


7. How we can define NON VALUE ADDING ACTIVITIES?
Activities that do not contribute to the product or the process and should therefore
be eliminated. Non-value added steps are waste.
8. Define VALUE ENABLING ACTIVITIES?
Activities or processes that do not, in themselves, meet the criteria of value adding
activities, but which must be performed in order to make it possible to perform
value adding activities.
9. What is PERCEIVED PRICE?
How much a customer thinks that a product should cost them? These perceptions
may or may not accurately reflect reality.
10. Describe ALIGNMENT.
It is the ability to have common and shared interests across the supply chain
including vendors and customers.
11. What is LOST SALE for a supply chain?
Sales you have lost because an item was out of stock or because you do not carry a
particular brand or line of merchandise. The customer is keen to buy the product.
12. Explain BULL WHIP EFFECT.

P
a
g
e
6
4

The bullwhip effect occurs when the demand order variability in the supply chain
are amplified as they moved up the supply chain. Moving up the supply chain from
end-consumer to raw materials supplier, each supply chain participant has greater
observed variation in demand and thus greater need for safety stock. The effect is
that variations are amplified as one move upstream in the supply chain (further
from the customer).
13. What is SUPPLY CHAIN PROFITABILITY?
Percentage of profit that a supply chain collectively creates is called as supply
chain profitability.
14. Define VALUE OF A SUPPLY CHAIN.
A value chain is a chain of activities for a firm operating in a specific industry. The
business unit is the appropriate level for construction of a value chain, not the
divisional level or corporate level. Products pass through all activities of the chain
in order, and at each activity the product gains some value. The chain of activities
gives the products more added value than the sum of the independent activity's
value. It is important not to mix the concept of the value chain with the costs
occurring throughout the activities. A diamond cutter, as a profession, can be used
to illustrate the difference of cost and the value chain. The cutting activity may
have a low cost, but the activity adds much of the value to the end product, since a
rough diamond is significantly less valuable than a cut diamond.
15. How should we measure CONCEIVED VALUE?
Value of the product after it has been used by the customer is the conceived value
of the product. Conceived value should also be increased.
16. Define Efficiency, Effectiveness and flexibility?
EFFICIENCY
Organizational efficiency is defined as an internal standard of performance and is
approximately a construct for doing the things right. From resource dependence
perspective efficiency is an independent measure for evaluating organizational
productivity.

P
a
g
e
6
5

EFFECTIVENESS
Organizational effectiveness is defined as an external standard of how well an
organization is meeting the demands of the various groups and organizations that
are concerned with its Activities which approximately is a construct for doing
the right things or having validity of outcome.
FLEXIBILITY
The ability to respond more quickly to demand and opportunities, flexible-
supply chains that can deal with the changes and preferably in a better way than
their rivals. In doing so, a competitive advantage can be achieved.
17. Define LOGISTICS?
Logistics is the management of the flow of goods between the point of origin and
the point of use in order to meet the requirements of customers or corporations.
Logistics involves the integration of
information, transportation, inventory, warehousing, material handling,
and packaging, and often security. Logistics is a channel of the supply chain which
adds the value of time and place utility.

18. Define SUPPLY CHAIN?
A supply chain is a system of organizations, people, technology, activities,
information and resources involved in moving a product or service
from supplier to customer. Supply chain activities transform natural resources, raw
materials and components into a finished product that is delivered to the end
customer.
19. Briefly define OPPORTUNITY COST.
Opportunity cost is the cost of any activity measured in terms of the value of the
best alternative that is not chosen (that is foregone). It is the sacrifice related to the
second best choice available to someone. The opportunity cost is also the cost of
the forgone products after making a choice.
20. When can say that a organization or process is STRATEGIC FIT?

P
a
g
e
6
6

It is when strength is capable to take advantage of opportunities. Strategic
fit express the degree to which an organization is matching its resources and
capabilities with the opportunities in the environment.
21. What is competition, competency, core competency ?
COMPETITION
Competition is a contest between individuals, groups, etc. for territory, a niche, or
a location of resources. It arises whenever two or more parties strive for a goal
which cannot be shared. Competition is usually for an advantage.
COMPETENCY
It is the ability or skill that will help the competitors to get advantage in
competition.
CORE COMPETENCY
Core competencies are particular strengths relative to other organizations in the
industry which provide the fundamental basis for the provision of added value.
22. Describe the situation of HYPER COMPETITIVE MARKET?
A situation in which there is a lot of very strong competition between companies,
markets are changing very quickly, and it is easy to enter a new market, so that it is
not possible for one company to keep a competitive advantage for a long time.


23. Explain VENDOR MANAGED INVENTORY.
Vendor Managed Inventory simply means the vendor (the Manufacturer) manages
the inventory of the distributor. The manufacturer receives electronic messages,
usually via EDI, from the distributor. These messages tell the manufacturer various
bits of information such as what the distributor has sold and what they have
currently in inventory. The manufacturer reviews this information and decides
when it is appropriate to generate a Purchase Order.

P
a
g
e
6
7

24. What is LEAD TIME for a supply chain?
The amount of time that elapses between when a process starts and when it is
completed. Lead time is examined closely in manufacturing, supply chain
management and project management, as companies want to reduce the amount of
time it takes to deliver products to the market.
25. Define BEST PRACTICES?
A best practice is a technique or methodology that, through experience and
research, has proven to reliably lead to a desired result. A commitment to using the
best practices in any field is a commitment to using all the knowledge and
technology at one's disposal to ensure success.

GENERAL QUESTIONS: TECHNOLOGY

Interview Question: Tell me about a time when it was necessary for you to
communicate technical information, clearly, to an audience.
Answer Guide:: Applicant should appear to keep current with current
technological advancements and they should be able communicate that information
succinctly and clearly.
Interview Question: What plans have you in the near future to take extra technical
training, and how will it help your career prospects?
Answer Guide:: Applicant should want to acquire new technical knowledge.
Interview Question: Give me an example of an assignment, which you have
recently worked on, that involved the learning of a new technical development.
Answer Guide: Applicant should not show resistance, but a willingness to learn.
They should want to understand and implement new knowledge.
Interview Question: How computer literate are you, and which software programs
are you familiar with?
Answer Guide: Knowledge of software should include word processing,
presentation software, database and spreadsheet programs, and industry specific
software

P
a
g
e
6
8

Interview Question: Could your work performance be improved by incorporating
new technical knowledge and developments?
Answer Guide: Job seeker should push for new technical training and to
implement this new knowledge into work practices.
Interview Question: What is your level of experience with software used in your
job? What resources do you use when faced with a PC problem?
Answer Guide: Candidate should be able, and prepared, to communicate a
difficulty to colleagues. They should be aware of online and software resources,
such as Microsoft help and support, to overcome difficulties.
Interview Question: Have you ever used software to make a work related
presentation?
Answer Guide: Applicant should have experience of making presentation and of
using PowerPoint.

GENERAL INFORMATION ON TECHNOLOGY
Q: What is your view of IT (Information Technology)?

A: Most companies are failing to take advantage of advances in IT and IT
departments are largely following rather than leading adoption. Web sites are
rarely used internally and e-mail remains the most popular document sharing tool.

Q: What percentage of the company's (or division's) budget is spent on IT issues?

A: I can't really answer that. I'm not hiding anything; I just do not know that level
of budgeting.


Q: How has the role of IT changed over the past decade?

A: In the late 1980s to early 1990s, there was a growth in IT tools as computers
were placed on every desk top. From the late 1990s to early 2000s, IT departments
have slowly adjusted to managing this vast amount of hardware and software that

P
a
g
e
6
9

is not in a centrally controlled room and may be distributed throughout a building,
an office park, a city, or nation-wide.
Q: How will, or how must, the role of IT change in the future?
A: IT must learn how to manage laptops and other mobile equipment, work with
non-corporate assets such as personal laptops and PDAs, enhance sharing of
information within the company and between companies, and lastly take on more
of a training role - many non-IT staff cannot take full advantage of word
processors, calendar programs, VPNs, and other corporate tools.

Q: How does the company (or division) manage knowledge?
A: Most information sharing is still done by e-mail and copy and paste between
systems. IT services are usually limited to restricting access to information.

Q: How do you use risk management?
A: We largely do risk management by the book. We have Excel spreadsheets that
take risk impact and risk likelihood and calculate risk level. There is a lack of
confidence in the system and we still tend to react only when problems occur.

Q: How does the company use risk management?

A: Each project reports level of risk as a green, yellow, or red indicator. Reporting
a red risk status usually requires writing additional status reports, so most project
managers avoid reporting anything besides green or yellow status. I am not sure
how upper level management uses risk reporting, but the only response I have ever
seen is increased requests for status.


Legal Aspects of Business

1. Define a contract.

P
a
g
e
7
0

Ans Contract is an agreement enforceable by law.
2. What is the competency of the parties to enter into a contract?
Ans A minor (less than 18 yrs of age), a person of unsound mind, and an
undischarged insolvent can not enter into contract
3. What is the main remedy for the breach or violation of a contract?
Ans- one can file a suit for recovery of damages in a civil court.
4. Define Partnership.
Ans- Partnership is entered between two or more than two persons (maximum
10 for a Partnership doing banking business and 20 in other cases) for carrying
on business and to share profits.
5. What is object of the Limited Liability Partnership Act, 2008?
Ans The object of the Limited Liability Partnership Act 2008 is to provide for
the registration of Partnership Firms with limited liability.
6. What are Negotiable Instruments?
Ans- Promissory Notes (P Notes), Bill of Exchange & Cheque.
7. Define requirement of complaint under Section 138 of the Negotiable
Instrument Act, 1881.
Ans- (i) Cheque drawn to discharge certain debt liability must have bounced on
account of insufficient funds.
8. Define a Company.
Ans- Company can be defined as an incorporated entity, having a separate legal
identity with perpetual succession, a common seal, a common capital comprised
of transferable shares and carrying limited liability.
9. What is the meaning of incorporation of a company?
Ans- The registration process of a company with the Registrar of a company is
known as incorporation of the company.
10. What is the difference between private and public company on the basis of
membership?
Ans- A private company has minimum 2 and maximum 50 members and a
public company can have minimum 7 and maximum any number of members.

P
a
g
e
7
1

11. How disputes in the Industry can be resolved?
Ans. The industrial dispute can be resolved by three methods, namely-
conciliation, arbitration and Adjudication ie through judicial proceedings.
12. What are the legal requirements for a valid strike?
Ans. In order to be a valid strike 14 days notice has to be served and this notice
remains valid only upto 6 weeks.
13. How much percentages of wages are to be paid for the striking workers?
Ans. As per the Supreme Court orders No work No pay is the rule.
14. What are the prerequisites in order to be eligible for the retrenchment
benefits?
Ans. There are following requirements for getting retrenchment benefits-
(i) The name of the worker should be borne on the muster rolls
(ii) He should be in continuous service (240 days in a year and 190 days in
underground work viz. mines etc
15. What is the rate of retrenchment benefits?
Ans. One is eligible to get 15 days retrenchment benefit for every year of
service rendered.


Economics

1) How best to define economics?
Economics is that branch of social science which studies how societies use their
scarce resources to produce commodities, and how the societies distribute the
produced commodities among different groups of people and over time.

2) Why do some people believe that a mixed economic system solves basic
economic problems?
It is because both the private sector and public sector have a say in answering the
basic economic questions, thus, there will be a degree of high efficiency (due to the
private sector involvement) and social welfare too (due to the public sector

P
a
g
e
7
2

involvement). It allows the Government to intervene when the economy faces
market failure. The mixed economic system allows markets to operate freely until
it fails to allocate resources efficiently, after this, the government agencies, such as
the EU Competition Commission, are able to correct for these failures. If this had
been a completely free economic system, Government agencies would not have the
means of power to be able to intervene.

3) What are the advantages of free market economy?
There are many advantages to a free market economy. Unprecedented innovation -
Free markets are wrought with inventions and the capital to research them.
Countries classified as having a free market have been responsible for the vast
majority of inventions since the 19th century. Very high-income mobility - This
means that under a free market system it is easier to move around income brackets.
It is just easier to become rich or poor when you are left to your own devices as
opposed to a controlled economy where resources are allocated by the government.

4) What are the disadvantages of mixed economy system?
The disadvantages of a mixed economy really depend on how "mixed" it is. For
instance, if it is mixed more towards a free-market, there is little regulation (some
may see this as a good, though), but if it is mixed more towards a command
economy, the control may stifle growth.
Mixed economies can also have different characteristics. Each of these will share a
different set of disadvantages. A will stifle profits due to it's high tax structure, but
will encourage new ideas due to its low regulation (this could result in many weird
effects such as an economy comprised almost solely of small, well-niche
businesses). B will encourage profits, but due to its regulation, some new ideas
(and some growth) will be stifled. For instance, if environmental regulations are
strict, the building of new plants or refineries might be lowered. This could result
in a small number of very large and profitable businesses.

5) What does macroeconomics mean?
The study of the overall aspects and workings of a national economy is such as
income, output, and the interrelationship among diverse economic sectors. It is the
study of all aspects of the economy. It is different from microeconomics, which

P
a
g
e
7
3

studies how individual entities (such as people, families, or even corporations) fit
in the economy.

6) What are the types of market economies?
There are 3 main types of market economies. The Free Market, Mixed Market, and
Command Economy. The Free Market, just simply means let the invisible hand in
the Economy work, while in the Command Economy, the invisible hand is the
government. The mixed Economy has a mixture of both.

7) What is a monopoly?
When one business or company dominates its area and squeezes out all its
competition, the result is the consumer does not have a free choice, and inevitably,
the price of its products or services will increase, and the 'Monopoly' increases its
profit.

8) What is an Oligopoly?
Oligopoly is a market where the supply is controlled by a small group of
companies. In this condition, the actions of one company will have a material
effect on the entire market for a product.
Several characteristics of an Oligopoly:
1) Substantial barriers to entry 2) Market dominated by a few large firms 3)
Differentiated products 4) Price rigidity
9) What Does Gross Domestic Product - GDP Mean?
The monetary value of all the finished goods and services produced within a
country's borders in a specific time period, though GDP is usually calculated on an
annual basis. It includes all of private and public consumption, government
outlays, investments and exports less imports that occur within a defined territory.
GDP = C + G + I + NX
where: "C" is equal to all private consumption, or consumer spending, in a nation's
economy "G" is the sum of government spending "I" is the sum of all the country's
businesses spending on capital "NX" is the nation's total net exports, calculated as
total exports minus total imports. (NX = Exports - Imports)

P
a
g
e
7
4

10) What are the functions of price mechanism in a free market economy?
Price mechanism is the point, which equilibrates supply and demand within a
market. It is a mechanism of pricing. The price mechanism is one, which allows
the prices of good and services to be decided by the interplay between supply and
demand. There is no centralized price fixing.
The price mechanism is the concept that the free market, when left to its own
devices, will formulate fair prices of the goods or services on its own by the natural
laws of supply and demand.
11) What is the impact on the economy if price ceiling or price floor were
removed?
Price ceiling is government rules or laws setting price floors or ceilings that forbid
the adjustment of price to clear markets. Price ceilings make it illegal for sellers to
charge more than a specific maximum price. Ceilings may be introduced when a
shortage of a commodity threatens to raise its price a lot.

12) What is meant by the term National Debt?
When a government spends more than it receives in taxes, it runs a budget deficit,
which is usually covered by issuing debt obligations to domestic and/or
international investors. In the US, these obligations are Treasury bills, Treasury
notes, and Treasury bonds. The total outstanding amount of such obligations
constitutes a National Debt.

13) What is inflation and how is it measured?
Inflation is defined as a sustained increase in the general level of prices for goods
and services. It is measured as an annual percentage increase. As inflation rises,
every dollar you own buys a smaller percentage of a good or service.
There are several variations on inflation:
Deflation is when the general level of prices is falling. This is the opposite of
inflation.
Hyperinflation is unusually rapid inflation. In extreme cases, this can lead to the
breakdown of a nation's monetary system. One of the most notable examples of
hyperinflation occurred in Germany in 1923, when prices rose 2,500% in one
month!


P
a
g
e
7
5

Stagflation is the combination of high unemployment and economic stagnation
with inflation. This happened in industrialized countries during the 1970s, when a
bad economy was combined with OPEC raising oil prices.
There are 2 types of measurement, CPI, and WPI. Consumer Price Index - A
measure of price changes in consumer goods and services such as gasoline, food,
clothing and automobiles. CPI is the most common measurement, using a market
basket of goods and measuring their price from a point in the past (a CPI of 100 is
arbitrarily the same price level for 1982-1984). Thus, the equation is (Price of most
recent market basket/price of same market basket in 1982-1984) X 100. The 100 is
to give us the number we normally see. WPI is Wholesale Price Index. It is a
measure of wholesaler's prices and is generally considered a pre-cursor to what
CPI will be (as it takes time for goods to read the consumer).
14) What is the difference between recession and depression?
Many professionals and experts around the world believe that a true economic
recession can only be confirmed if GDP (Gross Domestic Product) growth is
negative for a period of two or more consecutive quarters.
Before the Great Depression of the 1930s any downturn in economic activity was
referred to as a depression. The term recession was developed in this period to
differentiate periods like the 1930s from smaller economic declines that occurred
in 1910 and 1913. This leads to the simple definition of a depression as a recession
that lasts longer and has a larger decline in business activity.
A good rule of thumb for determining the difference between a recession and a
depression is to look at the changes in GNP. A depression is any economic
downturn where real GDP declines by more than 10 percent. A recession is an
economic downturn that is less severe.

15) State and explain 5 factors that affect a consumers demand schedule.
The factors that may influence a consumers demand schedule are -
The price of the product itself
Income of the consumer
Prices of other goods
Consumer tastes or preferences
Government regulations


P
a
g
e
7
6

16) A consumer, Ramesh, spends all of his income on 2 goods X and Y. The 2
goods are both normal but are not complementary. The price of good X is reduced
but the price of good Y is unchanged. The consumer continues to spend all of his
income on the 2 goods. Distinguish between the Substitution Effect and the Income
Effect of the price reduction in good X.
The Substitution Effect - as the price of good X falls the consumer buys more of
this cheaper good because of the law of equi-marginal returns, and the consumer
not being in equilibrium any more. When the price of good X falls the marginal
utility, for every rupee spent, on good X increases, and in order to maximize his
utility the consumer buys more of X. Demand for good Y falls because it is
relatively dearer.
The Income Effect A fall in the price of good X causes a rise in real income. As
both goods are normal goods demand for both X and Y increases.

17) Explain 3 assumptions concerning consumer behaviour.
The consumer acts rationally which means that he acts in a way that is consistent
with his personal preferences.
The consumer has limited income Because the consumer has not enough income
to satisfy all of his desires he must make a choice between those goods that he
wishes to buy.
The consumer aims to get maximum satisfaction from his income. The consumer
will obey the Equi-Marginal Principal of Consumer Behaviour.

18) A Company in Gurgaon has calculated the following Cross Elasticity of
Demand values for a number of its products as follows:
Cross Elasticity of Demand between Good A and Good B = + 5
Cross Elasticity of Demand between Good A and Good C = - 0.3
Cross Elasticity of Demand between Good A and Good D = + 0.6
Cross Elasticity of Demand between Good A and Good E = -2.9

19) What is the significance of the above figures? Include in your answer an
explanation of Cross Elasticity of Demand and the formula by which it is
measured.

P
a
g
e
7
7

Cross Elasticity of Demand measures the percentage change in the demand for one
good caused by the percentage change in the price of other goods. The formula is
Cross Elasticity of Demand for substitute goods is always positive the greater the
positive value the closer the substitutes. Goods B and D are both substitutes for
Good A, with B being the closest substitute.
Cross Elasticity of Demand for complementary goods is always negative e.g. if the
price of cars rise the demand for cars (eventually) falls and as a result the demand
for petrol falls. Therefore the demand for petrol and the price of cars move in
opposite directions. Goods C and E are complementary to A.
20) Explain the meaning of the words Elastic and Inelastic as they apply to Income
Elasticity of Demand?
Elastic means sensitive and inelastic means not sensitive. In the case of Income
Elasticity of Demand a product like milk is inelastic because if income increases
the quantity consumed of milk remains much the same. Milk has a low positive
Income Elasticity of Demand value e.g. + 0 .3
On the other hand a product like fruits is elastic because if income increases the
quantity consumed will rise substantially and it will have a high positive Income
Elasticity of Demand value e.g. + 3.

21) Iscon Ltd produce two goods that have income elasticity values as follows -
Good A - 0. 5 and Good B + 4, Which of these values is likely to represent income
elasticity of demand for foreign travel and why?
Foreign travel is very sensitive to income changes i.e. it is highly elastic. This
means that if income increases by 10% demand will increase by more than 10%
and it is likely to have a value of +4. So Good B represents foreign travel.
Good A is an inferior good as it has a negative income elasticity of demand
meaning that as income rises demand for this particular good falls.

22) It is assumed that Elasticity can be put to great use by producers in the decision
making process. Discuss this statement.
Producers use price elasticity of demand because it indicates the sensitivity of
demand to a change in price. Inelastic goods and services should be raised in price
and elastic goods and services should be lowered in price.


P
a
g
e
7
8

23) State and explain 5 factors which affect the foreign exchange value of the
Indian currency (Rupee).
The foreign exchange value of the Indian currency is the price of that currency in
relation to other currencies. Its value is determined by demand and supply. If
demand by foreigners for the Indian currency exceeds the supply of the Indian
currency then its value will rise and vice versa.
The main factors affecting its value are:
Indian Exports, Indian Imports, Interest rates, Market speculation and RBI
Monitory Policy.

24) Indian exports have risen, in recent years. Discuss the effects of this
development on the amount of borrowing/receipts by the Indian government. An
increase in the demand for Indian goods by foreign countries causes an
acceleration of economic activity in India. Companies must expand in order to
produce these extra goods for export. Therefore they need to hire extra workers to
allow increased production to take place. As a result of all of this extra economic
activity the government collects more revenue from higher income tax, indirect tax
and corporation tax receipts.

25) Explain the difference between the Balance of Trade and the Balance of
Payments on the Current Account.
The Balance of Trade is the difference between visible exports and visible imports
whereas the Balance of Payments is the difference between total exports (visible
and invisible) and total imports (visible and invisible).

26) What do you mean by the following: 1 Gross Domestic Product at Market
Prices. 2 Gross National Product at Market Prices. 3 Gross National Product at
Factor Cost. 4 National Income.
1 Gross Domestic Product at Market Prices is a measurement of the economic
activity (income earned) taking place within the state including the contribution
made by foreign factors of production operating in India. However it does not
include the economic activity of Indian factors of production operating overseas. It
includes indirect taxes but does not include subsidies.
2 Gross National Product at Market Prices is a measurement of the economic
activity of, or income earned by, Indian owned factors of production operating in

P
a
g
e
7
9

India and overseas. It excludes the income earned by foreign factors of production
operating in India. It includes indirect taxes but does not include subsidies.
3 Gross National Product at Factor Cost is a measurement of the income earned by
Indian owned factors of production operating in India and overseas. It excludes the
income earned by foreign factors of production operating in India, and indirect
taxes, but includes subsidies.
4 National Income is the income accruing to the permanent residents of the state
from current economic activity over a period of one year.

27) Explain the relationship between Gross National Product at Market Prices and
Gross National Product at Factor Cost.
Gross National Product at Market Prices less indirect taxes and plus subsidies
equals Gross National Product at Factor Cost
Indirect taxes are subtracted because Factor Cost is a measurement of the income
earned by factors of production and market prices include indirect taxes, which are
taken by the government and never become income for factors of production.
Subsidies are added because they are not included in market prices but are part of
the income of factors of production.
28) Explain the relationship between average cost and marginal cost in the short-
run. Average cost is the cost per unit whereas marginal cost is the addition to total
cost as a result of producing one extra unit.
When marginal cost is less than average cost then average cost is pulled down.
When marginal cost is greater than average cost then average cost is driven up.
When marginal cost equals average cost then average cost remains the same.
Marginal cost crosses average cost at its minimum point.

29) What is the difference between an internal economy of scale and an external
economy of scale? Give examples as part of your answer.
An internal economy of scale is a factor that causes cost per unit to fall as the firm
produces a larger number of units. Examples include the availability of bulk
buying discounts and the specialisation of the workforce. An external economy of
scale is a factor that causes cost per unit of the individual firm to fall as the
industry, in which the firm operates in, grows in size.

P
a
g
e
8
0

Examples include the establishment of specialised firms like Maruti Suzuki in
producing component parts for the larger firms and the provision of specialized
training courses for the workforce. Both of these lead to a fall in the long-run
average cost curve of the firm.

30) What causes the short run average variable cost curve to be the shape that it is?
The declining part of the average variable cost curve is caused by specialization of
the workforce which causes the productivity of each worker to rise. The increasing
part of the average variable cost curve is caused by the law of diminishing returns.

31) What are the assumptions underlying Imperfect Competition?
There are a large number of buyers and sellers.
Product differentiation exists i.e. it has been established in the minds of the buying
public, through competitive advertising and branding,
There is freedom of entry to and exit from the industry.
Each firm tries to maximise profits i.e. they endeavour to produce the level of
output where marginal cost is equal to marginal revenue and they aim to minimise
cost.

32) Give an example of an industry operating under conditions of an Oligopolistic
market structure.
An example of an oligopolistic industry is the retail petrol market. In this market
there are a few sellers such as IOL, HPCL, BPCL, Reliance and so on. The
products sold are very close substitutes with companies maintaining brand loyalty
by all sorts of product promotion schemes.

Questions on Quantitative Techniques
Q1. What is a variable? Give examples.
A. A variable is a quantity which takes different values in different situations
Examples :

P
a
g
e
8
1

-Family size/ No. of road accidents per day at Delhi/ No. of students present in
different Sections of an Educational Institution per day (These are examples of
discrete variables)
-Height/weight/profit of a company in different years/Production of rice in India in
various years (These are examples of continuous variables)
Q2. Define random variable
Ans.) Random variable is a real valued function defined on the sample space i.e. a
function whose range is the set of Real numbers or a subset of it and the domain is
the set of all possible outcomes of an experiment.
Q3. What is measurement ?
A. Measurement means assignment of real numbers to the object or event or a
property of an object by following certain rules.
Q4. Describe various levels of measurements.
A.Four level of measurements are
* Nominal(Classificatory) : Uniquely classifies - like classification w.r.t.
gender.
*Ordinal (Ranking) Preserve orders - Establishment of > or < relationship,
addition is allowed viz. average ranks
*Interval(no fixed zero point) like temperature. 60 c / 30 c is not equal to 2
i.e. ratios are not meaningful . It ensures Equal intervals and thus addition is
allowed
*Ratio (fixed zero point) - Natural zero - like measurement of length,
area, weight etc. where all mathematical operations are defined
Q5. What is frequency distribution of a variable?
A. A frequency distribution is a tabular summary of data showing the values (or
non-overlapping values of class-intervals) and frequency of each such
value(or class interval)

P
a
g
e
8
2

Q6.If frequency of a class-interval is 25%, corresponding angle of the class-
interval in the Pie diagram will be what degree?
A. 90 degree
Q7. If the variable is in percentages, which measure of central tendency is best ?
A. Geometric mean
Q8. For a grouped frequency distribution with few open ended class-intervals,
which measure of central tendency is not defined?
A. Arithmetic mean
Q8. What is the unit of measurement of Coefficient of variation ?
A.It is unit free
Q9.What does correlation indicates?
A.It indicates degree of linearity between two variables.
Q10. Differentiate between primary and secondary data
Primary data: information that is developed or gathered by the researcher or
investigator specifically for the project at hand i.e. data to be collected
Secondary data: information that have previously been gathered by someone
other than the researcher or the investigator for some other purposes i.e. data
which are available
Q11. From a sample if the largest value and the lowest value are eliminated then
which measure of central tendency will remain unchanged?
A. Median
Q12. If mean = median = mode then what could you say about the distribution?
A. The distribution will be symmetric
Q13.What is an event ?
A. An event is one or a combination of elements of sample space

P
a
g
e
8
3

Q14. Probability of an event is always a rational number. Do you agree or not ?
A. Yes, I agree since probability of an event is of the form m/n where m, n are
integers and n is not equal to zero.
Q15. If Probabilty that you will be selected in todays interview is 0.7, what is the
probability that you will not be selected today ?
A. 0.3
Q16.What do you mean by the statement that two events A and B are mutually
exclusive?
A. It means the two events A and B cannot occur simultaneously i. e. if A
occurs, B cannot occur and vice versa.
Q17. When do you say two events A and B are independent ?
A.If A and B are independent then probability of one event is not affected by
the occurrence of the other event. Two events A and B are said to be
independent if P(A|B) = P(A)or P(B|A) = P(B) or P(AB) = P(A) P(B)
Q18.Supose X takes values 0, 1, 2, 3, 4 and 5 with respective probabilities as p
1
,
p
2
, p
3
, p
4
and p
5
. What will be sum of P
1
+p
2
+p
3
+p
4
+P
5
?
A. One since we know that sum of all probabilities is equal to one.
Q18. Tell us two properties of a probability density function of a discrete
variable
A. If f(x) is the probability density function of a discrete variable X taking
values x
1
. X
2
, x
3
, and so on, then f(x) helps to find probability that X takes
any particular value i.e P(X=xi). Two properties of f(x) are
i) F(xi) 0 and ii) f(xi) = 1
Q19. Define expected value of a discrete random variable
A. Let X be a discrete random variable which takes values x
1
, x
2
, x
3
, .x
n

with respective probabilities p
1
,p
2
,p
3
,..p
n
. Then expected value of X
denoted by E(X) = p
i
x
i


P
a
g
e
8
4

Q20. Can you express mean and variance in terms of expected value ?
A. Mean of X = E(X) and Variance of X = E(x
2
) - [ E(X)]
2

Q20. Define Binomial distribution
A. Binomial distribution is a discrete probability distribution where a variable
takes values 0,1,2,3,n and probability density function is given by P(X
takes value r )=
n
C
r
p
r
q
n-r
where p is the probability of success in a single trial and p+q =
1
Q21. Mean is always greater than variance for which distribution?
A. Binomial distribution since we know that for a Binomial distribution mean =
np and variance = npq where p and q are non-negative fractions
Q22. When Binomial distribution is symmetric ?
A. Binomial distribution is symmetric when p=q= 0.5
Q23. Mean = variance for which distribution ?
A. Mean = variance is true for Poisson distribution which has a single parameter
called
Q24. What is a statistic ?
A. A statistic or a sample statistic is a function of sample observations like
sample mean, sample variance, sample proportion etc.
Q25. What do you mean by sampling distribution?
A.Sampling distribution of a statistic is the distribution of that statistic if measured
in various samples. Value of the statistic changes from sample to sample. Thus, if
there are k-samples ,the statistic can be viewed as a variable. Its distribution is
called sampling distribution of that statistic.
Q26.What are the components of a long time series data?
A.Components are Trend, Seasonal movements, Cyclical movements and
Irregular movements

P
a
g
e
8
5

Q27. Moving average method cannot be used for what ?
A.Moving average method cannot be used for the purpose of forecasting
Q28. For fitting a linear trend of the form Y = a + bt of a time series data where
t = 0, what will be the value of b ?
A. Value of b = yt / t
2

Q29. Explain Least Square method. Why the method is called Least Square ?
A. To fit Y = a + bX, we find values of a and b from the data such that
sum of squares of deviations of actual and predicted value of Y is least i.e.
(Y Yp)
2
is least where Yp denotes predicted value of Y
Since we are considering least value of a square quantity, the method is
called Least Square
Q30. What are normal equations ?
A.Normal equations are used to find values of parameters (unknown constants)
while fitting a curve to a data involving two or more variables including one
dependent variable. For example, to fit Y = a + bX, normal equations are
y = na + b x and xy = a x + b x
2

Q31. Suppose y = a + bt represents annual trend equation. What will be the
equation for monthly trend ?
A. Y = a/12 + bt/144
Q32.Probability of failure of Forklift Trucks (FLTs) made by company A and
Company B are found to be p
a
and p
b
in a sample. How do you know that the
probabilities are significantly different or not ?
A. Testing the hypothesis of equality of two probabilities in the population can
be viewed as testing a null hypothesis of equality of two proportions. The
same can be done by Z test.
Q33. What do you mean by normalization of a variable ?
A. Normalising a variable X means having a transformation (called Z-
transformation) as follows : Define Z
i
= (Xi -

)/SD(X) .Then for large



P
a
g
e
8
6

data, Z will follow a Normal distribution with mean = 0 and SD = 1 i.e.
standard normal distribution
Q34.Why two regression lines i.e. Y on X and X on Y are different ?
A. Regression equation of Y on X is Y = a + bx + error. Because of the error
term, the equation is not a mathematical equation but a statistical one.
Distribution of error for Y on X and X on Y are different. Thus, the two
regression lines are different .
Q35. What are the properties of two regression lines viz. Y on X and X on Y ?
A. Common point of the two regression lines is

. The angle between the


two lines will be small if absolute value of r
xy
is high
Q36. What is type I error ?
A. Type I error is unjust rejection of the null hypothesis i.e. error made if we
reject Ho when it is true
Q37. What is power of a test ?
A. Power of a test is 1 Probability of type II error where type II error is the
error made for unjust acceptance of Ho i.e. error for accepting Ho when it is
false.
Q38. What is critical value of test statistic ?
A. Value of the test statistic which separates the critical region and acceptance
region is called critical value of the test statistic.
Q39. What is level of significance ?
A. Probability of Type I error is called level of significance
Q40. Ordinate of Standard Normal distribution will be maximum at what value of
X ?
A. At X = 0 = Mean
Q41. How do you know that two attributed are associated ?

P
a
g
e
8
7

A. By computing value of chi-square statistic. Suppose the first attribute is
devided into p-classes and the 2
nd
attribute into k-classes. Then from p X k
matrix showing cell frequencies (i.e. observed frequencies), we find
expected frequencies under Ho ie if the attributes were independent and then
compute Chi-square = (Oi Ei)
2
/Ei Higher value of chi-square will
indicate that two attributes are associated.
Q42. If X follows normal distribution, then what will be the distribution of X
2

A. Chi-square distribution.
Q43. State null and alternate hypothesis of ANOVA
A. H0 : 1 = 2 = 3 . k
Where
k
represents population or treatment mean of the k-th Sample
null hypothesis indicates that - all means are equal,
Against the H1 : i j for at least one pair of i and j where i j
Alternative Hypothesis states that at least one pair of the means is different.
Q44. Average is not defined for which level of measurement ?
A. Average is not defined if the measurement is in Nominal and Ordinal scale
Q45. If X is a continuous variable and Y takes values 0 or 1 , then which type of
correlation can be defined between X and Y ?
A. Point bi-serial correlation

Questions on Operations Research

Q. 1. What is an Analog model ?
A. An Analog model is one in which one physical property is used to
represent another physical property
Q2. What do you mean by Decision theory models ?

P
a
g
e
8
8

A.Decision theory models are a class of Operations Research models designed to
select an optimal course of action from a set of alternative courses of action
Q3. What are Decision variables
A. Decision variables are the unknowns to be determined by solving the model
Q4. What is a Deterministic model ?
A.A deterministic model is one where the functional relationships and
parameters are known with certainty
Q5. What do you mean by Optimisation ?
A.Optimisation is achieving maximum or minimum or the best value of some
effectiveness criterion
Q.What is a Linear Programming problem ?
A.A linear programming problem LPP) is a constraints optimization technique to
optimize a linear measure of effectiveness (Objective function) under linear
constraints. It is concerned with the problem of allocating limited resources among
activities in an optimal manner
Q6. What are your ideas about Waiting Line theory ?
A.Waiting Line theory (also known as Queuing theory) deals with situations in
which queues are formed depending upon arrival rate, service rate etc. Helps to
have managerial decisions like number of service channels(or counters) etc.
Q7. Describe game theory.
A.Game theory is a class of Techniques for decision making in situations of
conflict
Q8. Can you say what is Operations Research ?
A.T here are large no. of definitions of O.R. each highlighting one or more aspects
of the subject. In short, it can be said that OR is a problem solving and decision
making science involving data gathering, building model, experimenting with

P
a
g
e
8
9

model, predicting future operations . It provides quantitative basis for decisions
regarding operations.
Q9. Define Slack Variable .
A. Slack variable is a variable added to the left-hand side of a less than or
equal to constraint to convert into an equality. It can be interpreted as
amount of unused resources
Q 10. What is a surplus variable ?
A. Surplus variable is a variable which is subtracted from the left-hand side of a
greater than or equal to constraint to convert the constraint into an equality.
This may be interpreted as the amount over and above the required minimum level.
Q11. What is ment by Basic solution of a LPP ?
A. For a system of m-simultaneous linear equations in n-variables (where n>m),a
solution obtained by setting ( n m ) variables equal to zero and solving for the
remaining m- variables is called a Basic solution of a LPP
Q12. What do you mean by Basic Feasible solution of a LPP?
A. A basic feasible solution of a LPP is a basic solution for which each m-variable
is greater than or equal to zero.. i.e. a basic solution which is feasible is called a
basic feasible solution.
Q13. What is Optimal ( or optimum) solution in LPP ?
A. A basic feasible solution which optimizes (minimizes or maximizes) the
objective function of the LPP is call an optimal or optimum solution
Q14. Clerify Unboundedness in LPP
A. Unboundedness is a condition describing LP maximization problems having
solutions that can become infinitely large without violating any constraints
Q 15. Describe Balanced Transportation problem .

P
a
g
e
9
0

A. A transportation problem in which total supply (from all the origins) is equal
to total demand (of all the destinations) is called a balanced transportation
pronlem.
Q16. What is an Assignment Problem ?
A. An Assignment Problem is a special type of LPP where the objective is to
minimize the cost or time of completing a number of jobs by a number of
persons in such a way that only one person is assigned to one job
Q 17. What is Maximax criterion ?
A. An optimistic decision criterion for dealing with uncertainty when probability
is not used. Here, one has to find for each option the maximum possible payoff and
select the one that produces the greatest maximum payoff








Q 18. What is the dual of a dual in LPP ?
A. Primal
Q.19. How do you know that you have reached the optimal solution to a LPP of
minimization type ?
A. If all Cj - Zj are greater than or equal to zero, then the current basic feasible
solution is optimal

P
a
g
e
9
1

Q. 20.What are Rim requirements if a Transportation problem ?
A. Total quantities of product which are supplied by each origin and the total
quantities demanded at each destination in the transportation method
Q.21. Name three methods of solving a Transportation problem
A. North-west corner Rule, Vogels Approximation method (VAM), and Modified
Distribution method (MODI)
Q 22.What is Hungarian method ?
A. A technique of solving assignment problems
Q 23.What is an unbalanced Assignment Problem ?
A. It is an assignment problem where the number of resources(or persons) is not
equal to number of activities ( or jobs)
Q.24. What is payoff in decision process ?
A. Each combination of a course of action and a state of nature is associated with a
payoff which measures net benefit to the decision maker that results from a given
combination of decision alternatives and events.
Q.25. What does EMV stands for?
A. Expected monetary value
Q.26. What is a decision tree?
A. A graphical device for analyzing decisions under risk. It illustrates the
chronological ordering of course of action and states of nature in a decision
analysis problem.
Q.27. What does MINIMAX criterion do in decision process?
A. This minimizes the maximum opportunity loss
.Q.28. Which criterion selects the action for which maximum payoff is lowest?
A. Minmax Criterion

P
a
g
e
9
2

Q.29. What is value of a game?
A. The expected payoff of game when all the players follow their optimal
strategies.
Q.30. What do you mean by a finite game?
A. A game is called finite if each player has the option of choosing from only a
finite number of strategies.
Q.31. How do you reduce a n X 2 game to a 2 X 2 game?
A. By using dominance rules i.e.by identifying dominant strategies separately for
each player or by using graphical method.
Q.32. If arrivals to a queue occur according to a Poisson distribution, and then
what will be the distribution of inter-arrival time?
A. Exponential distribution
Q.33. If customers arrive with an average time of five minutes between successive
arrivals, and then what is the arrival rate?
A. 12 customers per hour
Q.34. What can you infer if value of a game is negative?
A. The game is favouring one Player
Q.35. Suppose optimal strategy of Player A in a game is (0, 2/9, 7/9). What does it
mean?
A. It means that the Player A should never play the 1
st
strategy. Out of 9 plays, he
must play the 2
nd
strategy twice and the 3
rd
strategy for 7 times.
Q.36. How do you convert a maximization type Transportation problem to a
minimization type?
A. By subtracting each value of the given matrix from the largest value
Q.37. What will be the number of occupied cells of an optimal solution of a
Transportation problem?

P
a
g
e
9
3

A. m + n - 1 where m denotes number of rows and n denotes number of columns.
Q.38. What does FIFO indicate?
A. First in first out which is a queue discipline
Q.39. What is the relation between service rate and service time in Queuing
Theory?
A. Service rate and service time are reciprocal to each other
Q.40. How a game can be solved if it has no saddle point in terms of pure
strategies?
A. The game can be solved by considering mixed strategies i.e. a combination of
two or more strategies (for each player) selected one at a time according to pre-
determined probabilities.
.Q.41. What will happen if < where denotes arrival rate i.e. number of
customers arriving per unit time and denotes the service rate i.e. number of
customers served per unit time?
A. If < , then there will be
* No waiting time
* Utilization parameter will be less than one
* Service facility will remain idle for some time

Questions on Human Resource Management

1) What are human resources planning (or HR planning)?
Answer: HR planning links people management to the organization's mission,
vision, goals and objectives, as well as its strategic plan and budgetary resources.
A key goal of HR planning is to get the right number of people with the right skills,
experience and competencies in the right jobs at the right time at the right cost.
2) What is workforce analysis?
Answer: Workforce analysis involves identifying current and anticipated future
supply of labor and skills, identifying what you need and will need in the future in
terms of labor, skills and competencies (demand analysis), and then identifying the

P
a
g
e
9
4

gaps between the current and future supply and current and future demands (a gap
analysis).

3) What is the best HRMS (Human Resource Management System)?
Answer: Human Resources Management System is a computerized system that
accomplishes two very important purposes. First, it allows the keeping of
personnel data in a form that can be easily accessed and analyzed. In that respect it
functions as a means of collecting and organizing data about one's staff that can be
used as input into the HR planning function.
Second, it allows to the use of that data to fulfill the various functions usually
associated with a human resources department. The human resources management
software typically will help administer and record personnel functions through the
use of various software modules. A major strength of HRMS is that it is
comprehensive and can handle information and HR processes for almost every
possible organizational function, and that it's integrated and accessible through a
common computer interface. Integration allows exceedingly sophisticated data
management and reporting.
For example, a typical HRMS will include many or all of the following modules:
Payroll module
Recruitment, selection and applicant database module
Benefits administration module
Training and staff development module
Performance Management, appraisal and performance planning

4) What Is Succession Planning?
Answer: Succession planning is the process by which companies and businesses
ensure that there is an orderly and planned transfer of powers, responsibilities and
job functions when employees -- usually in executive and management positions,
leave the company through retirement, or other means. Succession planning
involves identifying positions and roles where vacancies are anticipated, and
identifying how the company will fill those positions.

5) How an HR Manager should conduct a performance review?


P
a
g
e
9
5

Answer: The purpose of a performance review is to provide a formal opportunity
for managers and employees to discuss strengths, weaknesses and areas for future
skills development. This should be through a two-way dialogue where both parties
learn how to better communicate and work together. A well-done review looks at
performance over the last period being measured, and will look toward future
objectives as well.

6) How can a company measure the recruiting costs?

Answer: A common measure is the cost-per-hire ratio. Depending on how your
company recruits, the numerator could include such costs as advertising, agency
fees, employee referral bonuses, applicant/staff travel, relocation, recruiter
pay/benefits, time involved, and other non-financial costs. The denominator would
be the number of people hired. This provides the company with a benchmark and a
control value.

7) What guidelines should HR manager consider before terminating an employee?

Answer: There are three major reasons why employees are terminated: Illegal or
improper behavior, poor job performance, and position elimination. When first
deciding that termination may be necessary, consider the impact on the
organization. How many employees will be affected, is the job loss permanent, will
there need to be a media spokesperson, are there any bargaining agreements that
affect this termination, etc.? Do not discharge an individual without properly
assessing the situations surrounding the termination. Once you have decided that
termination is probable, double-check your policies, procedures, and past practices
to maintain consistency. If the termination is affecting an employee from a
protected class, this is especially important. If the policies and procedures in the
past support a termination action, make sure all the appropriate documentation is in
place. At this time, make sure the assessment for the reasons of termination was
conducted thoroughly and that there is a paper trail supporting your decision.
Consult with an attorney, HR representative or consultant to determine any other
potential legal issues or risks. Finally, conduct a termination meeting in which the
reason for termination is clearly communicated to the employee and a member
from management and/or the Human Resource function present.

P
a
g
e
9
6


8) What forms does company need to complete while hire an employee?

Answer: The following forms are recommended to have a new employee complete:
Acknowledgment of Handbook, New Employee Information Form, Appropriate
Benefit forms, Receipt of Company Property, if applicable, Employment
Agreements, such as Confidentiality Agreements, Mandatory Arbitration
Agreement, Inventions Agreement, Internal Payroll Authorization Forms, New
Employee Orientation Checklist, Application for Employment (preferably filled
out prior to hiring) and a Job Description Acknowledgment.

9) How can a manager motivate his employees?
Answer: Some basic motivation guidelines: Tell people what you expect of them.
Be fair and consistent. Offer praise and credit for work well done. Give access to
yourself, your time, and your information. Know your people and respond to their
needs and perceptions. Provide interesting and challenging work - utilize their
talents. Get your employees involved. Give them a feeling that they are making a
contribution; that they make a difference.

10) Who should deal with an employee's grievance?
Answer: In most cases the employee's immediate line manager should deal with
the grievance, as he or she is likely to have an understanding of the issues. An
exception to this is where the grievance relates to the line manager, in which case it
should be dealt with by a different manager or someone from the HR department.

11) Are employee benefits taxable or non-taxable?
Answer: Medical insurance is taxable for employees. Any free equipment or
clothing used specifically in the course of work is not generally taxable, nor are
certain meals provided on the employer's premises or luncheon vouchers up to a
value of 15p per working day. Workplace nursery provision for employees'
children is not liable for income tax or national insurance contributions, provided
that certain qualifying conditions are met. Tax relief is available for certain
relocation expenses. Money given as a long-service gift is taxable.


P
a
g
e
9
7

12) How does an employer determine an employee's right to statutory maternity
pay?
Answer: If a woman is an employee, and is pregnant or has given birth, she is
entitled to statutory maternity pay provided that her average weekly earnings are at
least the lower earnings limit for national insurance purposes. She must also have
been continuously employed for a minimum of 26 weeks into the qualifying week
(which is the 15th week before the expected week of childbirth) and must have
stopped working for the employer paying her statutory maternity pay. Statutory
maternity pay is payable for up to 39 weeks.

13) What do you think are the roles and responsibilities of an HR Manager?
Answer: The purpose of this question is to understand your knowledge of the role.
The responsibilities of an HR manager include:
Recruitment, Training, Organizational Development, Communication,
Performance development and management, Salary and benefits, Team Building,
Building Employee relationship, Developing Leadership, Motivational exercises,
Developing companies policies and procedures, managing the health care program
for employees etc.
Depending on the size of the organization, the HR manager has a team performing
each of these roles. Knowledge, Intuition, Versatility, Flexibility, Focus and above
all a human touch in everything you do.

14) What is the difference between Personnel Management and HRM?
Answer: Usually these two terms are used interchangeably but there does lay a
difference between the two:
while HRM is proactive in nature and requires continuous development in policies
and functions of the organization for a better tomorrow.
ependent task than HRM which needs
to involve the managers of all departments for developing a better team.
management believes that satisfied employees yield better
performance while HRM believes that a good performance provides satisfaction to
employees.

P
a
g
e
9
8

employees. Personnel management works around motivating employees with
compensation, bonuses, rewards etc.

15) What do you mean by Management? Describe the essential functions of
management.
Answer: Management in simple terms can be understood as the process of bringing
people together to achieve the desired goals. The essential functions of
management include: Planning, Organizing, Budgeting, Resourcing, Directing,
Controlling and Reviewing.

16) What is the difference between salary and incentive?
Answer: Salary can be termed as the fixed remuneration you are offered for your
services while incentive means some extra benefits that you are offered based on
your performance. Incentive can be in cash or kind.

17) What do you mean by Fringe Benefits?
Answer: Fringe benefit can be defined as the benefits provided by the employer to
its employees in addition to the regular salary. These include health plans, paid
holidays, cafeteria, company transport etc.

18) Describe the ideal job.
Answer: Ideally, I would like to work in a fun, warm environment with individuals
working independently towards team goals or individual goals. I am not concerned
about minor elements, such as dress codes, cubicles, and the level of formality.
Most important to me is an atmosphere that fosters attention to quality, honesty,
and integrity.

19) What is the role of a trainer in a company?
The role of a trainer is to develop a competency and skill sets in an individual to
perform his/her effectively and efficiently in the work place. The trainer should
communicate to the trainees about what is expected out of training in a simple and
professional way.
The trainer plays a pivotal role from start to end of the training that includes the
following:

P
a
g
e
9
9



choosing the relevant training methods


evaluating the post training session

20) What are the key issues that should be addressed in the design, conduct, and
evaluation of training programs?

The following are the key issues to be addressed for a successful training program:

visual aids
Relevant training materials


visual aids


21) What are the three most common reasons why change in management
fails in most organizations?

Candidate should understand the dynamics of change in any form of organization
and be able to determine the problems of conflict and how they relate to the
change. Candidate should be a problem-solver and handle dilemmas and/or
conflicts effectively. They should recognize the potential problems that may arise
from a lack of attention and the inability or reluctance to change.

22) As you develop a strategic vision for your organization what are the five
key criteria that you should focus on?
Answer should include the five following key criteria: Organization; Observation;
Views (the environmental view; the marketplace view; the project view; and the

P
a
g
e
1
0
0

measurement view); Driving forces; and ideal position. The candidates ability to
define his/her ideal position in clear, strategic terms is plus.

23) How should you go about identifying allies as part of any good business or
organizational strategy plan?

Candidate should be flexible, be an influential decision maker on their own, and
manage good relationships with co-workers. For example, when groups with
similar interests create strategic alliances, they are much more likely to achieve
their goals. Allies may also be sympathetic insiders. A good candidate should
understand these concepts. A sympathetic senior bureaucrat in the right
organization who understands your project can also provide the most help. Finding
such a person and fostering that relationship shows initiative.
24) What is the basic difference between HRM and HRD?
A. HRM is a subset of the entire management processes of an organization. HRD
is a subset of HRM.
B. Scope of HRM is wider. Scope of HRD as compared to HRM is narrower.
HRM manages and develops the human elements of an organization in its
entirety on longer term basis. HRD focuses on those learning experiences which
are organized for a specific period to bring about the desired behavioral
changes.
C. HRM emphasizes that employees, their abilities and their attitudes constitute an
important organizational resource that should be used effectively and efficiently
to achieve organizational as well as employees' goals. HRD emphasizes mainly
on training and development of employees.
D. HRM takes decisions on HRD plans. HRD thus depends on the decisions of
HRM.
E. HRM at its center has HRD. HRD's cooperation is important for overall success
of HRM. HRD has to work within the realm of HRM and therefore, its

P
a
g
e
1
0
1

objectives should be in tandem with the broader objectives of HRM. Thus,
HRM and HRD are interdependent.
F. HRM takes care of all the human needs and tries to satisfy these needs so that
the employees are motivated from all the angles to contribute their best to
achieve organizational goals. HRD focuses on upgrading the skills and
competencies of the employees in order to improve the performance of the
employees on the job.

25) What is HR Score card? What is the importance of having HR
scorecard?
HR scorecard measures the HR function's effectiveness and efficiency in
producing employee behaviors needed to achieve the company's strategic goals.
So, HR scorecard is a measurement as well as an evaluation system for redefining
the role of HR as a strategic partner.
In order to achieve that organization would need to:
- Know what the company's strategy is
- Understand the causal links between HR activities, employee behaviors,
organizational outcomes, and the organization's performance
- And have metrics to measure all the activities and results involved.
There are seven steps required to develop HR score card, so that the result are
statistically measure
- define business strategy
- outline company value chain
- identify business required organizational outcomes strategy
- identify required workforce
- identify relevant HR system
- design of HR score card for measurement purpose
- a periodic revaluation based on measurement system from score card

P
a
g
e
1
0
2

The HR Scorecard offers the following benefits:
1. It reinforces the distinction between HR do-ables and deliverables
2. It enables cost control and value creation
3. It measures leading indicators
4. It assesses HRs contribution to strategy implementation
5. It lets HR professionals effectively manage their strategic responsibilities
6. It encourages flexibility and change

26) How would you relate the HR scorecard to Balance scorecard?
The Balanced Scorecard emphasizes the importance of measuring business
performance from the perspective of strategic implementation, rather than relying
solely on financial results. Senior managers tend to pay far too much attention to
the financial dimensions of performance and not enough attention to the driving
forces behind those results. Financial measures are lagging indicators i.e. backward
looking. They are designed to rectify or change past results. Performance drivers
on the other hand are within the control of the management in the present and the
Balanced Scorecard methodology encourages management to look at these leading
indicators as well. By specifying the important process measures, assessing them,
and communicating the firms performance based on these criteria to the
employees, the managers can ensure that the entire organization participates
actively in the strategy implementation process. HR Scorecard with the same
approach assesses the HR strategy and implementation in the organization.

27) What is basic difference between Performance management and
performance appraisal?
Performance Appraisal Definition

P
a
g
e
1
0
3

Appraisal systems were the precursor to todays Performance Management.
Appraisal systems are:
a) Typically based on a review of how a person completed their job for the year
b) Sometimes a pay review
c) Sometimes a review for bonuses
d) Sometimes an assessment of the employee for promotion
e) Typically conducted annually or less frequently
f) Typically paper based where HR is the custodian of the information. Otherwise,
they are conducted less formally and without any documentation
Performance Management Definition
Performance Management systems typically are:
a) A definition of what you want an employee to do for the next performance
period (normally the next quarter or half a year). The definition takes the form of
specific objectives for the quarter, backed up by a job description which takes into
account the normal expectations for that position
b) A review of remuneration/bonuses if these objectives are met
c) A review of the personal development objectives
d) Performed quarterly or half yearly
e) Typically fully automated where the information is accessible to all participants
at any time
f) Content rich if the automated Performance Management system has features
such as a Performance Diary
In simple terms performance management can be seen as a holistic process which
aims to bring together a number of aspects, including appraisal. Thus, performance
management may be thought of as being more strategic in its intent to achieve high
levels of organizational performance. By contrast, performance appraisal is best

P
a
g
e
1
0
4

seen as being more operationally focused, with a focus on individual employees
short-to medium-term performance and development

28) What is psychometric test? Name some good psychometric test and its
use.
Ans: Psychometric tests are concerned with the psychological measurement of
individuals. It includes the measurement of knowledge, intelligence, abilities,
attitudes, personality traits, and educational measurement. In the organizations,
these tests are used to provide employers with a reliable method of selecting the
most suitable job applicants or candidates for promotion.
Name of Some Psychometric Tests are:
MBTI (Myers Briggs Type Indicator) personality test, Big-Five personality test, 16
PF personality test, OPQ (Occupational Personality Questionnaire),
Psychometric tests are used for verbal, numerical and inductive reasoning, speed
and accuracy, motivation, personality and so on.

29) Why competency assessment is important when organizations have
performance appraisal?
Competency mapping is a process of identifying key competencies for a particular
position in an organization, and then using it for job-evaluation, recruitment,
training and development, performance management, succession planning, etc.
Introduction of competency mapping has also involved introducing skill appraisals
in performance appraisals. Companies have long realized the importance of
competency mapping as an important HR function. This is especially relevant in
this recessionary environment where human capital is one of the most important
assets of an organization and needs to be nurtured. Organizations always need to
have the best quality manpower. More and more companies are using competency

P
a
g
e
1
0
5

mapping tools to screen and hire people with specific competencies. There might
be a need for a person with excellent writing skills or an excellent creative mind.
When individuals must seek new jobs, knowing ones competencies can give one a
competitive edge in the market.

30) Name some good consultancies that are in core HR practices.
1. Hay Group
2. AON Hewitt
3. Mercer-Consultancy
4. Price Water House Coopers

31) What is the use of statistical tools in the area of HR? If given a chance
for which area you would use it.
There are a number of uses for statistics in human resources such as
1. Analysis of training data
2. Employees attendance and Discipline.
3. Employees Attrition data and business performance
4. Types of training and its effect on individual performance
5. Recruitment cost and employee tenure
6. Employees attitude and conflict management
7. Employees satisfaction and job involvement

32) What difference you can make out between employee attrition and
employee turnover?
Employee turnover and employee attrition both occur when an employee leaves the
company. Turnover, however, may result from a number of employment actions,
such as discharge, termination, resignation or job abandonment. Attrition occurs
when an employee retires or when the company eliminates his job. The major

P
a
g
e
1
0
6

difference between the two is that when turnover occurs, the company seeks
someone to replace the employee. In cases of attrition, the employer leaves the
vacancy unfilled or eliminates that job role.


Leadership and Teambuilding

1. What do we need to study leadership for the success of organizations?

Leadership is crucial for the success of all organizations because good
leadership helps to promote synergy, collect people and unify them, grow
interpersonal bonds, remove individual differences, restore team
cohesiveness and discharge roles effectively and efficiently.


2. Explain the concept of Leadership Pipeline in todays organizations?

Hiring an executive from outside is merely a short term option. For survival
and growth, an organization needs to develop a pool of successors to keep
the wheel of excellence moving. Managers at each level of the organization
take the responsibility to achieve business goals. In order to achieve the
goals, an organization must use a model of leadership development termed
as leadership pipeline. Example: GE s model comprises of 6 levels i)
managing individual contributions, ii) managing managers, iii) managing
functional managers, iv) being a business manager, v) being a group
manager, vi) being an enterprise manager.

3. What is legitimate or position power?

By virtue of the position of a manager in an organizational hierarchy and on
accepting the responsibilities, the person gains and enjoys his power. The
police, regulatory authorities like the pollution control boards,

P
a
g
e
1
0
7

environmental protection bodies, and many other forums have legitimate
power.

4. Why do leaders need Emotional Intelligence?
Emotional Intelligence is the intrinsic ability that helps an individual to
perform well under pressure, setting very high standards for him/her and the
team even in very tough situations. Therefore in order to be an effective
leader, emotional intelligence is inevitable.

5. Define Creative Intelligence.
Creative Intelligence is the ability to produce work that is both novel and
useful and it reflects how an individual connects the internal world to the
external reality.

6. Describe the managerial Grid of leadership Styles.
Blake and Mouton gave a two dimensional view of leadership, which is
referred to as leadership grid. The dimensions proposed in the grid are
concern for people and concern for production. In the managerial grid,
five different types of leadership based on concern for production (task) and
concern for people (relationships) are located in the four quadrants. The five
leadership styles are impoverished, country club, task, middle of the road
and team.

7. What is Fiedlers Contingency Theory?

Fiedlers Contingency theory centers on the belief that there is no best way
for managers to lead. Different situations create different leadership styles
requirements for managers. The style that works in one environment may
not work in another. He looked at three elements that dictate a leaders
situational control i) leader member relations, task structure, position power.

8. What is Transformational Leadership?

Transformational leadership is about change, innovation, and
entrepreneurship. This type of leadership is a behavioral process capable of

P
a
g
e
1
0
8

being learned and managed. Transformational leaders inspire followers, gain
their trust, exhibit such behavior that contributes to the achievement of
organizational goals, and perform at a high level.

9. Discuss the five stages of Group Development.

Tuckman (1965) propounded his Forming, Storming, Norming,
Performing, Adjourning model to explain team development and behavior.
He stated that as the team develops, its maturity, ability, and interpersonal
relationships also develop. Consequently the team leader also changes his
leadership style. In the Forming stage members meet and form a group,
members are confused about their role and the need of a team. In the
Storming stage members express ideas and there is an open conflict between
members. Sometime after the team formation in the Norming stage, the team
develops certain work habits that support group rules and values. Performing
stage is characterized by high levels of loyalty, participation, motivation,
concerted efforts, and group decision making. Leader becomes a facilitator.
Lastly in the Adjourning stage the group breaks up on completion of the
task.

10. What is a virtual team?

In an era of Information Technology, we are aware of teleconferencing-two
members located in US, three in Pakistan, two in Canada, and four in India
participating in a meeting. Instead of being geographically dispersed, they
work in the same company and take part in decision making. A virtual team
allows the members to meet without concern for space or time and enables
organizations to link the workforce together, which could not have been
done in the past.

11. A high performing teams leader should be assertive because..

Assertiveness is a social boldness that enables a person to express his/her
viewpoint without fear. An employee participates more boldly, takes on
greater responsibilities, expresses his/her ideas, and interacts with greater

P
a
g
e
1
0
9

variety of people, solves problems objectively, finds solutions and
implements them with minimum or no resistance.

12. What do you understand by Team Effectiveness?

According to Hackman (1990): Team Effectiveness is not a function only of
the teams ability to work together interdependently and effectively, but also
of the personal well being or satisfaction of the team members; the teams
output must also meet the standards of quality of the people who receive or
review that output.

13. How does empathy influence Team Effectiveness?

The most vital need while working in teams is mutual trust and as such this
dimension considers whether team members feel comfortable with each
other. Empathy is strongly related to the ability of a team to communicate
effectively regarding difficult issues without affecting the dignity of others.

14. What is the significance of value based leadership?

Value is indicative of worth, ethics, honour, and finally right and wrong
actions. Values are stable and enduring. Organizations today are realizing
the need for change in their management practices. Whatever be the
leadership style of a manager, effectiveness is possible only if the leaders
values emanate from the heart and not just from what he/she preaches. This
is value based leadership. By virtue of these values these leaders can foresee
the directions and speed of changes, understand people, and gain the
confidence of supporters.

15. What is the difference between a leader and a manager?

Leaders differ from managers. Manager controls activities, processes, keeps
track of activities, uses resources, plans and executes policies and runs the
business. A leader on the other hand created activities, processes, finds new

P
a
g
e
1
1
0

ways of performing, changes activities, finds resources and develops the
business.


Organization Behaviour II
1. What are the trait theories of leadership?
Theories that consider personal qualities and characteristics that differentiate
leaders from non leaders are known as trait theories. Example leaders like
Steve Jobs, Indra Nooyi, and Nelson Mandela are popular and well
recognized all over the world because they have traits like -being
charismatic, enthusiastic and courageous.

2. Who are transactional leaders?
Leaders who guide or motivate their followers in the direction of established
goals by clarifying role and task requirements.

3. What do you understand by conflict in organizations? State some of the
conflict management techniques.
Conflict is a process that begins when one party perceives that another party
has negatively affected, or is about to negatively affects something that the
first party cares about. Some of the conflict management techniques are
problem solving, super ordinate goals, expansion of resources, avoidance,
smoothing, compromise, etc.

4. What are the various steps in the process of negotiation?

The basic stages of any Negotiation process consist of the following:
A). Preparation: Identify all issues, set priorities, and develop support arguments;
b). Opening Session: Ground rules, framework for success, initial offers, posturing;
c). Bargaining: Single and Multiple issue negotiations;
d). Settlement: Impasse, third party interventions, closing.

P
a
g
e
1
1
1


5. Contrast Distributive and Integrative Bargaining.

Distributive Bargaining is a negotiation that seeks to divide up a fixed
amount of resources; it is a win-lose situation, whereas negotiation that
seeks one or more settlements that can create a win-win solution is known as
Integrative Bargaining.

6. Discuss the bases of power given by French and Raven.

Power is the capacity that A has to influence the behavior of B so that B acts
in accordance with As wishes. The various bases or sources of power have
been classified as under:
a). Formal Power: This is based on an individuals position in the
organization. It can come from the ability to coerce or reward, or from
formal authority.
b). Personal Power: This comes from an individuals unique characteristics
example Intels chip designers.
c). Reward Power: This comes from the ability to distribute rewards that
others view as valuable.
d). Expert Power: The influence based on special skills and knowledge.
e). Referent Power: Influence based on identification with a person who has
desirable resources or personal traits.

7. What comes to your mind when you hear organizational politics

Activities those are not required as part of a persons formal role in the
organization but that influence, or attempt to influence the distribution of
advantages and disadvantages within the organization are known as
organizational politics. Political behavior is outside specified job
requirements. It includes efforts to influence the goals, criteria, or process
used for decision making. Examples include withholding key information
from decision makers, joining a coalition, whistle blowing, spreading
rumors, leaking confidential information to media, etc.


P
a
g
e
1
1
2

8. What is organizational culture?

A system of shared meaning held by members that distinguishes one
organization from the other. Some of the primary characteristics that seem to
capture the essence of an organizations culture are: innovation and risk
taking, attention to detail, outcome orientation, people orientation, team
orientation, aggressiveness, and stability.

9. What is organizational climate?

The shared perceptions organizational members have about their
organization and work environment. This aspect of culture is like a team
spirit at the organizational level. When everyone has the same general
feelings about whats important and how well things are working, the affect
of these attitudes will be more than the sum of the individual parts. Example
psychological climate is strongly related to individuals level of job
satisfaction, involvement, commitment and motivation.

10. How do think organizational culture is transferred to employees?

Culture is transferred to employees in a number of forms, the most potent
being stories, rituals, material symbols, and language.

11. Identify the forces that act as stimulants to change.

Change or die is slogan these days among todays managers world-wide.
Some of the forces stimulating change are: nature of the workforce,
technology, economic shocks, competition, social trends, and world politics.

12. List some of the sources for resistance to change.
Our egos are fragile and we see change as threatening, as a result most of us resist
change. Some of the sources of resistance to change are:
Individual Sources-Habit, Security, Economic Factors, Fear of the Unknown and
Selective Information Processing.

P
a
g
e
1
1
3

Organizational Sources: Structural inertia, Limited Focus of Change, Group
Inertia, Threat to Expertise, and Threat to Established Power Relations.

13. Discuss Kurt Lewins Three Step Model of Change.

Kurt Lewin argued that successful change in organizations should follow
three steps: unfreezing the status quo, movement to a desired end state, and
refreezing the new change to make it permanent. The status quo is an
equilibrium state. To move from equilibrium to overcome the pressures of
both individual resistance and group conformity unfreezing must happen
one of the three ways. The driving forces, which direct behavior away from
the status quo, can be increased. The restraining forces, which hinder
movement away from equilibrium, can be decreased. A third alternative is to
combine the first two approaches.

14. Organization Development is a collection of change methods that try to
improve organizational effectiveness and employee well being. Explain.

Organization Development (OD) is a collection of planned change
interventions, built on humanistic-democratic values, that seeks to improve
organizational effectiveness and employee well being. OD methods value
human and organizational growth, collaborative and participative processes,
and a spirit of enquiry.

15. What do you understand by Action Research?

Action research is a change process based on the systematic collection of
data and selection of a change action based on what the analyzed data
indicate. Its value is in providing a scientific methodology for managing
planned change. Action Research consists of five steps: diagnosis, analysis,
feedback, action and evaluation.


P
a
g
e
1
1
4

Industrial Relations

1. Define industrial relations.
Ans- Industrial relations means relationship between management and
workers in the industry.
2. What are the major issues involved in industrial relations.
Ans- The major issues involved in the industrial relations are employment &
non-employment, terms of employment and conditions of labour.
3. Discuss basis of salary in traditional industrial relations and emerging
employee relations.
Ans- In traditional industrial relations the basis of salary was standardized
and in emerging employee relations the basis is individual performance.
4. What are the main sources of competitive advantages?
Ans- The three main sources of competitive advantages are cost, innovation
and quality leadership.
5. What are the approaches to the IR?
Ans- There are three approaches to IR, namely- unitary, pluralistic &
radical.
6. What is the objective of industrial relations?
Ans- The objective of industrial relations is maintenance of industrial peace
and high productivity.
7. What are the forms of IR?
Ans- There are three forms of IR, namely, management by contending,
management by conceding & management by colluding.
8. Discuss types of trade union.
Ans- There are two types of trade unions- blue collar trade union of workers
such as Hind Mazdoor Sabha, CITU etc, and white collar trade unions of
employers such as CII, FICCI, ASSOCHAM etc
9. Discuss strategies of conflict resolutions.

P
a
g
e
1
1
5

Ans- There are three strategies of conflict resolutions namely, lose- lose,
win-lose and win-win.
10. What are the different methods of conflict resolutions?
Ans- Collective Bargaining under non-statutory methods and conciliation,
arbitration & adjudication under statutory methods are the main methods of
resolution of the conflict resolution.

Cross Cultural Management

Why is Culture Important?
When intercultural trainers explain culture, many use the iceberg model. Simply
stated in everyday interaction we see only the surface level of culture, i.e. art,
books, film, food, fashion and language. On the whole these are very unlikely to
cause any great problems in the business world. It is the underbelly of the cultural
iceberg that poses the problems. These are areas such as attitudes, beliefs,
assumptions, world views, communication styles and values. If people are unaware
of such differences when it comes to such fundamentals, the potential for
misunderstandings, poor communication and failed relationships are increased.
Intercultural awareness training provides people with an insight into those very
differences and provides strategies to help overcome potential obstacles. One of
these obstacles when thinking about the Middle East is the approach to
relationships in business.

Does Culture Really Matter?
There are numerous concepts describing culture and how it inuences our behaviour. The most
popular is the iceberg model, in which we can see that only some aspects of culture are visible
to us and many more are hidden. Like an iceberg where we can only see the top
reaching out of the water, culture only shows us a few visible aspects and hides most of it
under the sea (or in our case in societies that we visit or encounter).Culture can best
be described through three main concepts: values, attitudes, and behaviour . All three signify
culture and allow us to differentiate from other cultures. The cultural orientation of a

P
a
g
e
1
1
6

society reects the complex interaction of values, attitudes, and behaviours
displayed by its members.

Values and attitudes shape and inuence actions and behaviour of human beings. They cannot
be seen and understood easily. The visible parts of culture are behavior, language, symbols,
rituals, and artifacts.

What is Cross-cultural misunderstanding?
Cross-cultural misunderstandings are based on the fact that we only see each others
behavior, but not the attitudes and values that explain it. If a Western person with individualistic
values and attitudes openly contradicts other team members or even his or her boss, this may be
considered very rude and inappropriate by an Asian person who often has more group-
oriented values and attitudes and would rather not speak up to avoid possible
conict. In most cases only behaviours or messages are exchanged, whereas values and
attitudes are hardly communicated. International managers can inadvertently make mistakes
when communicating with foreign business partners or employees.


Cultural Barriers
Research has shown that 20-30% of companies have identified a lack of cultural
awareness as an obstacle to doing business internationally. Although a high figure
it is important to question whether this can really be accurate. A crucial point to
consider is how companies have identified when cross cultural differences have
become a problem, i.e. if they don't appreciate that cultural differences can in the
first place cause problems, is it reasonable to believe that they could identify them
when they occur? It could very well be that the figure is lower than in reality.
Among the areas companies identify as posing particular cross cultural barriers are:
. Body language
. Etiquette
. Establishing trust
. Religious beliefs
. Social habits
Other more business related cross cultural barriers included differing approaches
to:
. Invoicing and payments

P
a
g
e
1
1
7


. Credit terms

. Customer preferences

. Packaging - i.e. sizes, colours, typeface
All the above mentioned areas are aspects of doing business which differ from
culture to culture and if there is a lack of synergy between parties it is often hard to
create an environment conducive to profitable business. It is clear from the results
of the study that those businesses that are unaware of how culture can play a role in
their business dealings abroad can suffer.

Intercultural Awareness
Intercultural awareness training is sometimes perceived as a luxury within the
business world. Many who feel this way are also proponents of the misconception
that because the "world is getting smaller" our differences are becoming less
obvious and less of a problem. This is in fact false. As we get closer, our
differences become more acute and more of a problem thus further driving the
need for intercultural awareness training.
Companies are now tuning into the fact that in order to get ahead in today's global
economy, cultural awareness is critical. It is no longer a luxury but a necessity. The
right intercultural awareness training briefing can offer a company real benefits,
namely a competitive advantage and a greater long term gain.

Potential Culture Clash
Now let's look at the potential areas for UK and Middle Eastern culture to clash.
1. "Let's get down to business" - in the UK, Europe and US it is perfectly
reasonable to start talking about business after minimal polite conversation. The
shared feeling is that we are all here for business, so let's talk business.
In the Middle East however business and personal life are not separated. Getting to
know people on a personal level is crucial in order to assess whether a relationship
can be established. As such if one were to jump straight to business without
investing some time in a "getting to know you" process you will be seen as
uninterested, inflexible, rigid and lacking sensitivity.

P
a
g
e
1
1
8

2. Personal Space - personal space is respected in the UK. People speak to each
other at a distance and touching is kept to a minimum. However in the Middle East
this is the opposite. Once should be prepared to be held, touched, felt and kissed!
Recoiling from such behaviour would make one seem cold.
3. Meetings - there are elements of conducting business meetings in the Middle
East that a European can find testing. For example, people may take telephone
calls. For the European this is seen as slightly rude whereas for the Arab to not
answer the call is rude! You may be in a meeting a find that people walk in and out
of the meeting room, will sit down next to you and completely change the topic of
conversation. For the European this is a distraction but for the Arab it is simply
hospitable to allow people in, and those that wander in will naturally want to speak
with you. It is important to always be patient in meetings and to allow for delays
and distractions.
4. Negotiations - negotiations can be lengthy and frustrating affairs. There are
quite a few elements someone should bear in mind when considering negotiations.
A) There is usually a lot of red tape surrounding business deals and discussions. B)
Arabs often make decisions on gut and heart feelings, not on facts and figures. If
they are unsure about you they will want time to try and learn more about you. C)
Decisions usually come from the top. If you are not negotiating with the decision
maker you are most likely wasting your time and simply being sussed out. D) If the
right level of trust has not yet been established through the building of a
relationship then this may be holding up proceedings.
5. Disagreements - Due to the need to save face in this part of the world it is never
a good idea to be too direct with people. When disagreeing one should as
diplomatic as possible. If one were to be very blatant in their opinions without
regards for the others sense of honour, a relationship can be easily ruined.
6. Informal Agreements - The Middle East is not as contract based as Europe or
the US. In fact much of the time someone's word is considered enough. This can
make those unused to this way of doing business nervous. To become insistent on
contracts and deadlines can make Arabs become rather stand-offish as they would
consider such behaviour as signifying a lack of trust in them.

Building Relationships
The following are some pointers to the main areas one should consider if they are
serious about building strong and profitable relationships in the Middle East:

P
a
g
e
1
1
9

1. Invest time - in order to build a relationship you must be willing to invest time.
Do not expect deals to be completed in two visits to the region. One must spend
time nurturing the relationship and show great awareness as to when it is at a stage
appropriate to take things forward in a business sense.
2. Small talk - informal chatting is critical. Never jump straight into business at a
meeting. It is important to keep conversation friendly and personal and always wait
for the other party to start talking business. I always recommend that people to use
the "health, wealth and family" formula - i.e. ask about their personal well-being,
business and the kids.
3. Get Personal - one should not be afraid to open up and divulge personal details.
In order to really get a relationship at the stage where the trust is firm and mutual
both sides need to get personal.
4. Understand face - one must observe and try and understand how face, shame
and honour work in the region. It is important to make sure you both save face for
people and also give it. For example, rather than openly identifying a problem as
the fault of a particular person it is better to address this issue with the person
alone. They will appreciate that you have helped save face. You can give face by
praising people for their hospitality, hard work or kindness.
5. Using networks - in the Gulf region there is a concept known as "wasta". This is
where people use their connections in order to get things done. For example, if a
person needs a visa to Sweden but have left it too late, they can see if people
within their network system have "wasta" in the embassy. If they do, they will ask
them to help get them the visa. In order to succeed in the region it is important to
use wasta and to also reciprocate favours done for you.
6. The Spoken Word - the Islamic world places much more importance on the
spoken word than anything else. This goes back to the beginnings of Islam where
the Quran was a revelation disseminated through speech. The first adherents of
Islam accepted the religion through trust in the Prophet Muhammad and his word.
Again this goes back to the whole idea of honour and shame, i.e. if a person was to
lie this would bring shame upon them. Therefore it is critical that one keeps to their
word. Broken promises will lead to broken relationships.
7. Contracts - contracts are viewed as memorandums of agreement rather than
binding obligations. One can still have contracts signed but never see them as
water tight. If in a business relationship both sides started to communicate through
a contract it is a sure sign that the relationship has broken down. In the Middle East

P
a
g
e
1
2
0

problems and disagreements should not be sorted out by pointing to clauses but
rather through personal discussion.
8. Continuity - once a relationship has been built it is important to maintain
continuity. If there is to be a personnel change in the region it is critical that the
person leaving the region introduce the new face. This allows the local contacts a
sense of continuity and importantly the reassurance that their new contact is
someone they can trust and rely on.
9. Evolution - the Middle East is steeped in religion, culture and a pattern of
thought. It is important to always remember that things change slowly there.




Relationship Driven Cultures
The Middle East is what we would call in intercultural jargon a "relationship
driven culture", i.e. personal relationships form the basis of social (and business )
interaction. Relationship driven cultures usually have the following traits:
1. Collectivist - this means that in such cultures the "we" takes precedence over the
"I". This group mentality means the interests, opinions and decisions of the group
carry much more weight than that of the individual.
2. The Family - the family or tribe takes central focus in daily life. In such cultures
very tight relationships are built with a small group of people whereas in more
individual cultures people tend to have loose relationships with many people. Such
family centred cultures tend to put the interests of the family first. Manifestations
of this are that nepotism is seen as natural and protecting the honour of the family
is a very high priority.
3. Hierarchy - a hierarchical society it used to levels of authority. A good example
of how a hierarchical society differs to a more level one is in management styles.
In less hierarchical cultures a subordinate is expected to use initiative, share in the
decision making process, can say "no" to the boss and most of the time has an
informal relationship with the boss. In hierarchical societies the boss takes sole
control because that is what they are paid for. Staff will expect explicit orders and
guidance, meetings will be where decisions are implemented rather than discussed
and very formal relationships exist with the boss.

P
a
g
e
1
2
1

4. Honour/Shame/Face - In relationship driven cultures there is usually an
emphasis on maintaining face, i.e. upholding the family/tribal honour. As a result
there are usually very complex rules of engagement and communication styles. For
example in the Middle East, saying "no" or blatantly disagreeing with people is not
usually done in order to save people's face. We therefore see a lot of "beating about
the bush" as people try to phrase sentiments in a way that does not make someone
lose face. A simple example would be that instead of "no" you may get "I will try",
"Let's do our best" or "God willing".
5. Networks - due to such cultures relying on bonds and relationships, networks
are usually the way things get done. An intricate means of favours and
reciprocation are part of daily life, from being introduced to the right people or
getting past red tape. Being part of a network gives you access to resources.
6. Consensus - in hierarchical societies decisions are usually made on a group
basis. Although in the Middle East final decision making is usually made by the
head of the family or tribe, there is still a level of consultation with others called
"shura". Shura means surveying the opinions of those who are most knowledgeable
in order to reach a decision that is best. Therefore within the business world it is
important not to only concentrate on building relationships with decision makers
but also those that advise them.

Why Cross Cultural Awareness?
"...the single greatest barrier to business success is the one erected by culture."
Edward T. Hall and Mildred Reed Hall
Working, meeting, dealing, entertaining, negotiating and corresponding with
colleagues or clients from different cultures can be a minefield.
Understanding and appreciating intercultural differences ultimately promotes
clearer communication, breaks down barriers, builds trust, strengthens
relationships, opens horizons and yields tangible results in terms of business
success.

What topics are covered in your cross-cultural courses?
Here are some examples of possible topics:
- Cross cultural awareness-raising programme
- Welcoming and hosting foreign customers (the country of your choice)
- Working with foreigners (the country of your choice)

P
a
g
e
1
2
2

- Preparing to move abroad: living and working abroad (the country of your
choice)
- Managing your return to France
- Helping impatriates to integrate in France
- Cross-cultural team-building
- International negotiation
- The virtual teams
- International public speaking
- International marketing
- Cross cultural aspects in Human Resources management
- etc.

What is the purpose of cross-cultural training?
Cross-cultural courses give your staff a better understanding of the culture they
will be immersed in, allowing them to adapt better to the new work environment
and thereby accomplish their mission more effectively.

To what extent do cross-cultural issues affect business?
According to many specialists, 60 to 80% of expatriation failures are due to staff
being ill-prepared for cross-cultural issues: incomprehension between the different
cultures in a company; difficulty adapting in occupational and personal terms;
difficulty "interpreting" the speech and acts of foreign colleagues
And a world where business is increasingly connected to the global community, an
understanding of other cultures can be as valuable as capital. A deep understanding
of how others value and conduct business helps not only business relationships, but
human relationships as well.

What is the difference between cross cultural training, intercultural training
and diversity training?

Sometimes the differences are more in semantics where different words can be
used to convey the same meaning. Cross-cultural usually refers to training that
involves increasing awareness about two or more cultures, perhaps where the
trainee is moving from one culture to another and the training will focus on home
and host cultures. Intercultural training often refers to training which increases

P
a
g
e
1
2
3

awareness between a number of different cultures, perhaps where the trainee will
be managing a culturally varied group. Diversity training is more often used where
the different cultures are present working together in the same workplace or
represented among the client base and focuses on increasing awareness and value
of the other cultures. Keep in mind that your trainers use of the different terms
may be more influenced by their own cultural background (Intercultural is a term
used more in the USA than in other countries for example). Ask questions to
determine what the content of the training will cover and what the outcomes should
be for your trainees. Trans Cultural Careers can provide training to cover any of
these areas of training need.


How is diversity training different from multicultural, cross-cultural, and race
relations training?
Multicultural training focuses on educating people to understand and appreciate
cultural differences. Diversity training focuses on building community rather than
pointing out how people are different. Appreciation of differences is important, but
it is not considered the highest priority competency. The abilities to make others
comfortable and included are most important, no matter how much you know
about their culture.
Cross-cultural training focuses on educating people to manage themselves in other
countries or as a minority in another cultural group. Diversity also includes gender,
sexual preference, religion, and other types of diversity that are not central in
cross-cultural training.
Race relations training focuses on educating people to understand and appreciate
racial differences, and helping different races get along. Race refers to skin color
differences in the United States while Europeans tend to include nationality in their
definition. American race relations trainers often assume that White American
privilege is a central problem in addressing racism. Diversity training includes race
relations while addresses the general problem of dealing with people who are
different. Institutional barriers are considered without placing blame on
individuals.

Can a diversity trainer create more problems that solution?

P
a
g
e
1
2
4

Yes, if the trainer is not competent. Most diversity trainers do not have credentials
other than a couple of workshops they attended and a strong desire to earn a living
in the area. A major source of incompetence results from the trainer's inability to
understand how their own biases create obstacles for effective training. Usually,
major problems are absent in the profession, even when people are incompetent.
However, some people exposed to diversity training experience adjustment
difficulties or heightened prejudice from poor training outcomes. These problems
occur when the trainer fails to offer opportunities for participants to state their
beliefs and values honestly or treat honestly with disrespect.

What Can Managers Do?
Managing diversity, like all effective management, depends on the ability to plan,
coordinate, be informed, identify opportunity, manage risk and build knowledge.
Diversity itself does not benefit an agency. Managers need to recognise, encourage
and develop diversity.
Managing diversity involves:
- providing thoughtful and informed team leadership
- being willing to learn, to adapt to difference and be inclusive of a diverse
range of work styles
- ensuring all staff understand and work to key organisational/team values and
practices
- fostering consultative and representative decision making and problem
solving
- identifying areas of need and supporting staff to improve performance
- considering diversity in functional and business decisions
- identifying and capitalising on diversity to foster adaptability and innovation
within the agency such as finding ways to transfer and better use staff skills
and experiences
- developing cross-cultural skills, awareness and competence in staff to
improve business operations and customer service.

What strategies can assist with language issues in the recruitment process?
Assess language skills The level of language expected at an interview needs to
match the requirements for the position. If particular language skills are considered

P
a
g
e
1
2
5

necessary for a job, obtain assessment advice from language professionals prior to
the interview and, if necessary, develop a language assessment tool.
During the interview, use the level of communication skills required to do the job.
This will lessen the applicant? language-related fears.
- Avoid bias toward accent
Take care to prevent possible bias toward accent. A subjective assessment of a
candidate's accent may indicate 'hidden' attitudes on the part of the assessor.
- Develop questions that are free of jargon or cultural bias
When developing questions for the selection panel, consider the style, length and
clarity of each question. Avoid jargon or agency-specific language. Establish the
degree to which questions can be rephrased.
- Ensure all questions, tests and screening tools are free from cultural bias.
Be aware that cultural difference may mean that questions are answered using a
variety of different approaches.
- Ensure questions, tests and screening tools are free from cultural bias.

What issues can arise in workplace communication?
Workplace reform has put more emphasis on teamwork, liaison, using information
technology, multi skilling, performance management and training. All these
demand an increase in the quality of communication skills of staff in the Public
Sector.
Good or bad, workplace communication involves everybody in the agency, and
everybody has a role in developing and improving it.
Too often, English language skill is the only communication issue considered
within the context of cultural diversity. The broader issue of everyone's inter-
cultural competencies and of the responsibility of the agency to develop this is
often not addressed.
Around 25% of the community in NSW speak languages other than English. In
day-to-day dealings in a multicultural society, Public Sector agencies are required
to respond to the diversity of languages that exist.
Language barriers and cultural misunderstandings can get in the way of effective
communication and create complexities in the workplace, including problems with
safety. It is important for managers and supervisors to gain expertise in cross-
cultural communication.

P
a
g
e
1
2
6


Understanding Common Cultural Barriers
Roles and status
A social hierarchy exists in most cultures, which can create issues in workplace
communications. For instance, men from some cultural backgrounds may not adapt
easily to working equally with women or having female supervisors.
Understanding the relationship between the sexes and the roles assigned in various
cultures will give you an insight into how staff from particular cultural groups
might respond in an Australian work context.
Differences in non-verbal behaviour
Cultural differences in non-verbal behaviours are a common source of
misunderstandings and conflict in the workplace. For instance, many westerners
like to make eye contact, interpreting it as an indication of interest and honesty.
They also show friendliness through relaxed body language. If these behaviours do
not happen, it can be interpreted as shiftiness, coldness and disinterest. However,
in some cultures, averted eyes and non-demonstrable behaviour are a sign of
courtesy and respect. Being aware of these nuances will help you to reduce
negative impressions in inter-cultural communications among your staff.
Personal space
People from different cultural backgrounds may have different 'comfort zones' -
some like more distance, some like to get closer.
Responses to authority
Attitudes to authority vary markedly between cultures and exert a distinct effect on
workplace behaviours.
Religion
In many cultures, religion dominates life in a way that for strongly secular
workplaces may be difficult to understand. These differences need to be respected
and the needs associated with religious commitments understood and sensitively
negotiated within the workplace. Clear communication on both sides about these
issues is to be encouraged.
Personal appearance
Grooming, eating habits and dress varies in different cultures. Some people within
the Australian workplace may choose to wear their national dress or religious
garments. Occasionally this could be unsafe or problematic. Handle these issues
sensitively and proactively to avoid unnecessary conflict and distress.

P
a
g
e
1
2
7

Keep in mind that attitudes and communicative behaviours are not neutral but
strongly conditioned by cultural values.

Managing Safety
When your workforce includes people from different backgrounds, managing
safety effectively requires a sharp focus on communication and training. As a
matter of duty of care, ensure that you have occupational health and safety
(O&HS) communication and training strategies in place for staff from culturally
diverse backgrounds.
Staff with limited English language skills are at greater risk of injury. Agencies
need to be careful not to underestimate the number of staff whose safety is affected
by language and cultural factors.
Tips for Improving Workplace Safety
To improve workplace safety:
- in every position, specify the English skills required for staff to work safely
- ensure that all staff understand verbal or written safety information
- ensure safety staff know how to communicate to a culturally diverse
audience
- check that all instructions, signs, labels and procedures are complete, written
in plain English and easily understood
- provide access to interpreters and translators where necessary.

Why is it important to provide learning and development for staff?
Fair and transparent access to training and development is critical for all staff,
including those from diverse backgrounds. Learning increases staff confidence,
participation and contributions of their representative perspectives.
Increasing, the levels of diversity in agencies are challenging educational practice.
Good practice means these processes need to be modified or reviewed in order for
them to be more equitable and inclusive.

Why is it so important to establish a workplace that is free from
discrimination and harassment?
Everyone has the right to a harassment-free workplace. Discrimination and
harassment affect all staff and all staff have a role to play in creating a workplace

P
a
g
e
1
2
8

culture that operates in an atmosphere of trust, respect and open communication.
This is the most effective means of harassment prevention.
Assisting supervisors and staff
Ensure that supervisors can distinguish between issues of process and those of
discrimination and harassment. Where staff are concerned that the guidance they
are providing to others could be interpreted as discrimination or harassment,
confusion and negativity may result.
Firm and determined management should not be confused with bullying and
harassment and supervisors need to be trained in the differences between the two.
Assist supervisors to build their confidence by offering training in:
- anti-discrimination and preventative harassment
- bullying processes in the workforce
- cross-cultural competence, particularly in managing staff performance.
Assist staff by:
- providing clear communication about processes and regulations
- checking that all staff are aware of these and understand them
- providing access to agency policies on grievances, harassment and bullying.
Harassment
Harassment is a form of unlawful discrimination. 'Harassment is defined as
unwelcomed or unreciprocated behaviour that makes an employee or customer feel
belittled, intimidated, offended or apprehensive.'5 Under the Anti-Discrimination
Act 1977 (NSW), it is unlawful to discriminate in employment or in providing a
service on the grounds of:
- a person's sex (including pregnancy), race, disability, marital status, age or
sexuality (homosexuality, transgender, bisexuality)
- a person's presumed or perceived disability or sexuality
- the sex (including pregnancy), race, disability, marital status, age, or age or
sexuality (homosexuality, transgender, bisexuality) of their associate or
relative
- the perceived or presumed disability or sexuality of their associate or
relative.
Harassment, such as physical assault, can be a criminal offence.
Problems are more likely to arise where these measures have not been put in place
in a workplace. For example, problems with process could be interpreted in

P
a
g
e
1
2
9

another context as exaggerated or generalised assumptions about the background,
race or language ability of some staff.
Workplace discrimination and harassment can cost agencies in lost productivity,
low morale, personal suffering and loss of agency reputation and service provision.
Additional financial costs can be incurred if staff decide to take legal action
because of discrimination and harassment.

Why promote diversity in the workplace?
Answer: According to the International Personnel Management Association's
Benchmarking Committee, "diversity efforts in the workplace facilitate the
exchange of new perspectives, improve problem solving by inviting different
ideas, and create a respectful, accepting work environment. Surveys have
demonstrated a positive impact on high performance where leadership teams
include a diversity of ages, ethnicity, and gender. A diverse workforce also can
improve organizational productivity and creativity. While managing a diverse
community can be a challenge, there is also potential for great accomplishment.
The key for employers is to make diversity an asset within the organization.

Why is diversity an issue for organizations today?
Answer: The Canadian labour market is undergoing deep, fundamental shifts due
to an aging population, a growing demand for highly skilled workers, and an
increasingly diverse population. By the year 2016, approximately two thirds of the
Canadian population aged 15-64 will be made up of people from the four
designated groups targeted by the Federal Contractors Program (women, visible
minorities, Aboriginal persons, persons with disabilities).







P
a
g
e
1
3
0

Questions on Negotiation and Counselling Skills
1. What is Negotiation? Discuss the reasons for Negotiations.

Negotiation is a means of getting what you want from others. It is a back and
forth communication designed to reach an agreement when you and the
other side have some interests that are shared and others that are opposed.
There are various reasons for negotiations like
a) To agree on how to share or divide a limited resource, such as land,
property or time;
b) To create something new that neither party could do on his or her own; or
c) To resolve a problem or dispute between the parties.


2. Why is it advised that before entering into any negotiation, one should
always clearly decide upon your BATNA?

Preparation for a negotiation process begins with the identification of key
goals-that is what exactly do you want or expect from the process. But
before entering into any negotiation, it is advisable to always clearly decide
upon your BATNA (Best Alternative To a Negotiated Agreement), or walk
away value. Without a clear BATNA, you start off in a weak position
because you lack a distinct reference point and thus may accept a poor offer
(one with a value less than if you achieved no agreement at all).

3. Differentiate between Distributive and Integrative Bargaining.

Distributive bargaining is a negotiation method in which two parties strive to
divide a fixed pool of resources, often money, each party trying to maximize
its share of the distribution. It is a fixed sum game, and the limited resource
is often termed as fixed pie. It is also called a zero sum process because one
party loses whatever amount is gained by the other.

Integrative bargaining on the other hand is a cooperative approach to
negotiation or conflict resolution. It is often referred to as a win-win or

P
a
g
e
1
3
1

mutual gains approach. This approach, like distributive bargaining, involves
making concessions to reach an agreement-but in addition, it involves
searching for mutually profitable options and logical trade-offs. It is also
called expanded pie approach because negotiators search for better proposals
than the obvious ones that meet only their interests.

4. Negotiation strategy is defined as an overall approach chosen when
confronted with a bargaining situation. Discuss.

We have found that most new negotiators jump right into their first
negotiation session without having given much thought to an overall
strategy, or they look for assistance from someone because they are not sure
how to start a negotiation process. The term strategy and tactics are often
confused and/or used interchangeably. What one experienced negotiator
might think of as her primary strategy in approaching a negotiation, another
might just as easily refer to as his primary negotiating tactics. Experts say
strategy comes first: a negotiator must know what the objectives are. and
develop a broad negotiation strategy.

5. Illustrate the five characteristics of Emotional Intelligence that relate to a
bargaining situation.

Goleman promoted the idea that for individuals to negotiate effectively, they
need to identify and use their Emotional Intelligence. Emotional Intelligence
encompasses
i) Self-awareness: the ability to be aware of which emotions one is
experiencing and why, as well as being aware of the effect ones
emotions are having on others,
ii) Self-regulation: the ability to keep ones emotions in check, to remain
calm in potentially volatile negotiations,
iii) Self-motivation: the ability to remain focused on ones goals in the
negotiation despite setbacks,
iv) Empathy: the ability to understand the feelings being transmitted at
the negotiations through verbal and non verbal messages, to provide
emotional support when needed, and

P
a
g
e
1
3
2

v) Managing relationships: the ability to deal with problems without
demeaning the opponents, not to let others negative feelings prevent
collaboration, and to handle conflict with tact and diplomacy.

6. What is Perceptual Distortion?

In any given negotiation, the perceivers own needs, desires, motivations,
and personal experiences may create a predisposition about the other party.
This is cause for concern when it leads to biases and errors in perception and
subsequent communication. Example: stereotyping.

7. What is Counselling? Discuss its essential elements.

Counselling is a method of identifying practical solutions to life and work
related problems. It is a means by which one person helps another through
purposeful communication. Given below are the factors that are essential to
the process of counseling:
a) An empathic relationship;
b) The counselor and the client relate well;
c) The counselor sticks closely to the clients problem;
d) The client feels free to say what they like;
e) An atmosphere of mutual trust and confidence exists;
f) Rapport is essential.


8. How can counseling be a source of organizational change?

Employers are turning to counselling as a way of helping the employees to
cope with the changes taking place in organizations. Change is never easy-it
disrupts, disorients, causes anxiety and takes time. Support is needed for
individuals and teams as transitions in organizations are managed.
Counseling is a way of supporting employees as they reel under the pressure
of organizational change.

9. Discuss the essential elements of Humanistic/ Person centered Counselling.

P
a
g
e
1
3
3


Carl Rogers is the founder of this Humanistic or Person centric counselling.
According to him, in any kind of psychotherapy the basic theme is helping
behavior. In all human interactions including mother child, teacher pupil,
manager-subordinate, counselor-client, the helping relationship is
fundamental. This relationship is intended to facilitate the growth of the
person receiving help. This growth is referred to as psychological growth
and is aimed at improving their functioning and/or accelerates their maturity.
Counselling helps in bringing about psychological maturity in their client.

10. What are the five steps in a counselling process?
Counselling has a predictable set of stages that occur in a sequencethey
are
A) Develop relationship
B) Define problem
C) Determine goals
D) Decide plan of action
E) Do follow up

11. What is the difference between Counselling and Psychotherapy?
Counselling deals with normal individuals and intends to guide an individual
towards better life, its goals are limited and focus more on the immediate situations
as helping the individual function adequately in appropriate roles. Whereas
psychotherapy deals with abnormal individuals and intends to prevent the
occurrence of mental or emotional breakdown, it is usually more vast and hard to
attain in comparison to .counselling.
12. What are the various approaches to Counselling?
a). Psychoanalytic Approach
b). Behaviouristic Approach
c). Humanistic Approach/Person Centric Approach
13. Briefly discuss the major goals of Counselling.

The following are the major goals of Counselling:
a). Facilitating Behaviour Change;

P
a
g
e
1
3
4

b). Improving Relationships;
c). Enhancing Coping Skills;
d). Promoting decision making; and
e). Facilitating the Clients Potential.

13. It is said that everyone cannot become a good Counsellor. Comment.

The National Vocational Guidance Association (1949) proposed that a good
counselor must possess the following characteristicsa deep interest in
people and patience with them, sensitivity to the attitudes and the actions of
others, emotional sanity and objectivity, a capacity for being trusted by
others, and respect for facts. Some of the other qualities which a counselor
must have include belief in each individual, commitment to individual
human values, alertness to the world, open mindedness and understanding of
self and professional commitment.

14. Why is initial rapport building an important aspect of the process of
counseling?

During the initial stage the counselor must create an appropriate atmosphere
and try to establish a special rapport with clients. This safe environment
encourages them to open up more, to take a closer and more objective look
at them, and ideally to challenge themselves in a way they might not
otherwise do. The conditions that induce clients to open up in such a way are
referred to as proper counseling attitudes. Further the counselor should have
unconditional positive regard for the client.

15. Discuss Termination of a Counselling Session.

The counselor will need to prepare the client for the end of the counseling
session. He/she should let the client know when the session is coming to an
end. As the session comes to an end, the counselor may ask the client to
summarize the main themes, feelings or issues of the session. This helps
clients crystallize in their own minds the important things that took place
during the session.

P
a
g
e
1
3
5

Compensation and reward management

Q: How is Compensation different from Payroll?
A: The Compensation staff studies and recommends the pay level assigned to UTD
positions based on the duties and responsibilities of those positions. The Payroll
staff calculates and issues payroll checks based on hours worked, deductions and
taxes.
Q: When are University employees paid?
A: Full-time, salaried university employees are paid once per month; hourly
employees are paid twice per month.
Q: What is a Pay Grade?
A: A Pay Grade is a designation attached to many classified job titles that share the
same salary range; for example both a Secretary III and a Locksmith are positions
in Pay Grade 7, as well as other positions. Administrative and Professional
positions do not have established Pay Grades.
Q: What is a Salary Range?
A: A salary range is a range of pay, determined by HRM Compensation, for each
classified position. There is a minimum rate, a 1/3 point, 2/3 point and a maximum
rate. Typically employees are hired or promoted into the first third of the range,
and as they grow in their position, they progress to higher salaries within the salary
range.
Q: What is a J ob Code?
A: Each job title assigned by HRM has a corresponding code that is required in
much of the HRM paperwork that is completed by departments. Also known as
Title Code, or Job Class code, Job Code is the correct terminology.

P
a
g
e
1
3
6

Q: How do you decide what salary to offer to a new hire or a promotion.
A: The annual Budget Policy communicates the amount and percentages
determined for increases for the fiscal year.
Q: What is a Merit Increase?
A: A merit increase is a performance-based salary increase granted to an employee
whose performance and productivity is consistently above that normally expected
and required. The current performance appraisal for the employee must be on file
in Human Resources Management (HRM). Merit increases may be granted only if
the employee has been employed for at least six continuous months before the
effective date of the increase, and the effective date is at least six months after the
employees last promotion, transfer, or merit salary increase. A Merit increase
should not be confused with a Cost of Living increase.
Q; what are the basic components of reward & compensation?
A: The reward or compensation people receive for their contribution to an
organization includes monetary and non-monetary components. Remuneration
does not simply compensate employees for their efforts - it also has an impact on
the recruitment and retention of talented people.
Q: What do you understand by fixed level of pay and pay linked to
performance?
A: There are two basic types of pay schemes, although many organisations have
systems which include elements of both:
- Fixed levels of pay. Wages or salaries which do not vary from one period to
the next except by defined pay increases, generally on annual basis. There may
be scales of payments determined by age, responsibility or seniority. Most
'white-collar' jobs were paid in this way until recently.
- Reward linked to performance. The link may be daily, weekly, monthly or
annualized. Payment for any one period varies from that for any other period,
depending on quantity or quality of work. Sales functions are commonly paid

P
a
g
e
1
3
7

on the basis of turnover; manual and production workers may be paid according
to work completed or items produced. Catering staff typically rely on direct
payment from satisfied customers in the form of service charges or tips
(gratuities).
Q: What do the terms Rewards, Total Rewards and Rewards Strategy mean?
A: Reward is often used interchangeably with the term compensation to refer to the
monetary components of an employees pay. However, reward has a broader
connotation to it. It is the term used to convey what an employee receives, in
monetary and non-monetary terms for demonstrating high levels of performance.
Hence reward is the tool used to recognize, appreciate and promote high
performance.
The new approach to define the remuneration of employees which calculates the
package in terms of the value to the employee is the Total Rewards
approach. Total Rewards is a mix of monetary and non-monetary rewards offered
to employees. Therefore Total Rewards comprises of Base Salary, Cash
Allowances, Bonuses and Incentives, Benefits (statutory and non-statutory),
personal and professional development opportunities, training and a motivating
work environment.
A Rewards Strategy is the reward philosophy or vision that an organization builds
in alignment with its business strategy. It is the fundamental approach that the
organization uses to communicate to its employees which behaviors it values most
and the kind of rewards programs that it plans to undertake to encourage
demonstrated examples of such behaviors.
Q: How effective is a differentiated rewards approach in driving performance
and maintaining employee engagement?
A: That is the way forward in order to reduce the sense of entitlement related to
performance pay that exists within employees.
In a heated talent market retention employees is of high priority and therefore
organizations take the safer route of minor differentiation in pay increases and
bonuses. While such an approach works effectively in overall retention, it does not
work effectively for retention of high performers.
Retaining high potential employees who are the business drivers is the only way to

P
a
g
e
1
3
8

remain on the growth trajectory that organizations have projected for themselves
and clearly differentiating them from the average performers can have huge
retention results.
In the context of the Total Rewards approach, organizations have also started to
understand that high performing employees have higher motivation levels due to
recognition. Such employees often have a high drive for achievement and hence
recognizing that they are role models or examples within the organization is
critical for them to consistently perform at those standards. Therefore, a clear
message on how the top performers are rewarded and the gap from the mediocre
performers is the direction to take. It is true that the largest population will be in
the latter basket, however, in order to achieve significant growth there is a need to
identify and reward suitably, especially in the current situation where reward
budgets are tighter.
Therefore in a heated talent market and highly competitive market, differentiated
compensation might not be the only retention tool employers have begun
focusing on differentiated recognition. Studies also show that recognition can exert
a powerful impact on employee performance (Stajkovic & Luthans, 1997; 2001).
Q: When providing salary increments or hiring, is it critical to focus on
market competitiveness solely or also look at internal equity?
A: There is a new area of concern that Reward professionals in India are now
facing lack of internal reward parity. A highly market driven approach especially
in a rapidly changing business landscape has led to internal consistencies in pay,
for individuals with similar job sizes. Benchmarking with external market practices
followed by successful organizations is an effective way to remain
competitiveness. However, conducting this exercise without consideration for a
suitable salary band framework within the organization leads to de-motivation. An
employee, who is aware of being paid lower than his or her peer with the same job
role, will be less productive and will have a disengaging effect on the entire team.
When referring to pay parity, the reference is to Base Pay which is aligned to the
size of the job. In order to manage minor differences in similarly sized jobs, salary
bands are created.
For example, conducting a broad based salary increase exercise using market
indicators keeps the firm competitive. However, hiring decisions dependent on
current market pay or solely driven by individual, without applying the principle of

P
a
g
e
1
3
9

robust internal parity, leads to friction.
Hence a pure market pricing exercise to determine pay leads to drastic gaps in
internal alignment and pose fundamental challenges in maintaining fairness and
transparency. The most important first step is to ensure that the jobs have been
analyzed and sized correctly. It is also important to focus on paying for the job and
not allow pay practices to be driven entirely by individual attributes. The guiding
principle should be that if a job size is bigger, has higher contribution and business
accountability then it warrants higher pay.

Q: How important is variable pay in driving performance?
A: Currently variable pay forms a small portion of the overall reward mix in India,
except for certain job functions such as Sales where it form a large part of the
salary by way of incentives and commissions. However the fundamental change
that has taken place in the past years is that in several organizations there is a
uniform annual short term variable pay plan covering all employees ( all
management levels and across functions). That is a remarkable change because
firstly it recognizes that each function has a business contribution direct or
indirect and secondly, it reflects the changing mindset in the direction of pay for
performance.
By designing a variable pay plan that comprises of measurable and valid
performance criteria results in-
1. Creating a sense of individual responsibility within employees for the
organizations progress.

2. Communicating that long term sustainable performance from employees will
have monetary as well as non-monetary rewards.

3. Driving the values that the organization stands for for example, does it
encourage teamwork through a team performance based incentive plan or does it
promote on individual excellence.

P
a
g
e
1
4
0


4. Re-aligning the mindset of perceiving performance linked pay as entitlement

5. Strengthening employee belief that since the organization is focusing on
investments in pay-for-performance, there is an objective performance
measurement process in place.

6. Positioning the organization as a market competitive player which attracts
individuals who are achievement oriented and confident of their capabilities to
deliver.
Q: How does one measure the effectiveness of a Rewards Strategy?
A: To measure the effectiveness of a reward strategy the organization has followed
a structured process of reviewing it and comparing the actual results of the
implemented strategy against the desired results. Such an evaluation is an
extensive but fruitful exercise. There are two sources to obtain information for
such an evaluation.

1. Employee feedback Often times organizations do not capture employee
feedback on how they have interpreted the strategy, its implementation and the
value they associate with it. The information to be captured is not whether there is
satisfaction related to current compensation levels, but whether they have a clear
understanding of the organizations philosophy and its implications for them. For
example, when driving a Total Rewards approach, employees should be made
aware of their entire package and not only cash compensation. Research has
clearly shown that employees in many organizations are unaware of or do not
understand the benefits offered by their employer (Dreher, Ash, & Bretz, 1988).
When employees lack this understanding, they tend to be dissatisfied with the
benefits.

2. Industry best practice There are other organizations with similar DNA who
may have implemented a strategy that is comparable. While it may not be feasible

P
a
g
e
1
4
1

to conduct a detailed comparison, a broad overview of which reward elements have
high returns and acceptance, is essential.
3. Define Outcomes At the onset, it is necessary to define clearly measurable and
demonstrable outcomes that are expected from this strategy.
In order to measure effectiveness early on in the strategy, some organizations may
consider a pilot or experiment implementation of the same for some sections of
identified employees but that may not be a feasible approach. With such a short
time-frame it is difficult to receive accurate results and there maybe disengagement
incase of employees who are not part of the experiment. Hence the only way to
find out whether the strategy is working or not is through actual implementation.
Those organizations which are agile, flexible and decision making is quick are the
ones which can revisit their strategy if it is not entirely effective.
Q: How does one balance the cost-investment reward approach?
A: In the past, India has experienced a situation wherein due to unprecedented
economic growth and an unbalanced demand-supply equation in the talent pool,
organizations have hired at significantly high salaries, some of which were aligned
to the job value and linked to accountability but some were not.
That kind of reward trend is primarily driven by external factors and has little
emphasis on job size. As a result of that trend, aberrations are created within the
organizations and their pay lines become significantly higher than competition, due
to a talent bidding war! In addition to that a huge pressure gets built within the
overall budget. In the wake of another impending upswing, organizations are
approaching the scarce talent pool with caution while their approach is shifting
towards how communicate the value of the package to the employee,
simultaneously they are closely tracking how their employee reward spend is
resulting in performance and whether this is aligned to the overall business
direction. Therefore treating reward as a cost leads to disengagement and
organizations should not focus on how to curtail it. Instead the emphasis should be
to contain and optimized it. It is definitely an integral and large part of the overall
budget hence while it should be treated as an investment but one which must be
capitalized and wisely reviewed.
In order to effectively maintain this balance, organizations must define parameters
which can be used to review the results of financial and non-financial reward
programs periodically and bringing in the concept of ROI within their purview.

P
a
g
e
1
4
2

An example of a balanced cost-investment approach would be our salary increases.
The total increment typically comprises of inflation, merit and promotion
increases. Providing an inflation-based increase only across the organization is a
reflection of reward being viewed as a cost. However, an inflation based increase
along with a merit increase, specifically for high potentials is a cost-investment
approach, where the merit increase conveys the importance of performance. It also
allows for measuring ROI by capturing reliable performance data.
Therefore while continuing to maintain the approach of reward being an
investment, organizations need to be cognizant of how it impacts their revenues,
employee productivity and retention, by designing better ROI measures and
assessing actual spend realistically
Q: What do you understand by person oriented pay strategy?
A: A strategy that pays individuals according to their market value requires a pay
strategy which measures the knowledge, skills and competencies of individuals and
prices them in the external market. So, organizations can pay individuals according
to their market value. They often can get a good idea of the market value of their
key individuals simply by writing job description and monitoring the actual hiring
transactions that go on in the labor market. This, of course, is a far different
approach from the traditional one of looking at salary survey data in order to gain
information about what individuals are paid in a particular job. And such pay
strategy will motivate workers to stay with company for a longer time and work
more effectively. Also, when organizations pay according to the knowledge of
workers, they will try to learn duties of other positions and become a key employee
which leads to the following outcomes. First, they will have a positive effect in
working area and good working conditions. And second, they will be paid more as
they become more valuable in the external labor market.
Q: What do you understand by position oriented pay strategy?
A: As Virginia and Polytechnic Institute (2010) says Provide market competitive
compensation that incorporates both salary and non-salary benefits, such as
healthcare, retirement, life insurance, disability insurance, annual and sick leave,
and other optional benefits... in order to assist formal job pay structure. This kind

P
a
g
e
1
4
3

of formal pay structure helps to group similar jobs together and it makes it easy to
pay for different job positions.
As Armstrong S. (2003) states, there are two most common forms of pay structures
which are Grades and Bands. Grades typically have very narrow range
spreads; while Bands have very broad range spreads. This has led to the commonly
used term of broadband to describe the range of pay rates within a structure
using a band approach. Thus Band has no midpoints. The Band pay structure is
much more flexible than it is in a Grade structure. Banding strategy is the
compression of a hierarchy of pay grades or salary ranges in to a small number of
wide groups. The aim of grouping is to provide easy pay structures, support
operational and team working, reflect the new emphasis and reward workers easily.

You might also like